Exam 4 (Assessment 4) Adult Health II

Pataasin ang iyong marka sa homework at exams ngayon gamit ang Quizwiz!

Which clinical findings are consistent with sepsis diagnostic criteria? Select all that apply. 1.Urine output 50 mL/hr 2.Hypoactive bowel sounds 3.Temperature of 102° F (38.9° C) 4.Heart rate of 96 beats per minute 5.Mean arterial pressure 65 mm Hg 6.Systolic blood pressure 110 mm Hg

3.Temperature of 102° F (38.9° C) 4.Heart rate of 96 beats per minute 5.Mean arterial pressure 65 mm Hg

Silver sulfadiazine is prescribed for a client with a burn injury. Which laboratory finding requires the need for follow-up by the nurse? 1. Glucose level of 99 mg/dL (5.65 mmol/L) 2. Platelet level of 300,000 mm3 (300 × 109/L) 3. Magnesium level of 1.5 mEq/L (0.75 mmol/L) 4. White blood cell count of 3000 mm3 (3.0 × 109/L)

4. White blood cell count of 3000 mm3 (3.0 × 109/L)

A client in shock develops a central venous pressure (CVP) of 2 mm Hg. Which prescribed intervention should the nurse implement first? 1.Increase the rate of O2 flow 2.Obtain arterial blood gas results 3.Insert an indwelling urinary catheter 4.Increase the rate of intravenous (IV) fluids

4.Increase the rate of intravenous (IV) fluids

Mafenide acetate is prescribed for a client with a burn injury to the hand. Which should the nurse include in the instructions to the client regarding the use of this medication? 1.If stinging occurs, discontinue the medication. 2.Apply a thinner film than prescribed to the burn site if the medication stings. 3.If local stinging and burning occur after the medication is applied, notify the health care provider. 4.It is normal to experience local discomfort and stinging and burning after the medication is applied.

4.It is normal to experience local discomfort and stinging and burning after the medication is applied.

The nurse is monitoring the fluid balance of a client with a burn injury. The nurse determines that the client is less than adequately hydrated if which information is noted during assessment? 1.Urine pH of 6 2.Urine that is pale yellow 3.Urine output of 40 mL/hr 4.Urine specific gravity of 1.032

4.Urine specific gravity of 1.032

A client sustains a burn injury to the entire right and left arms, the right leg, and the anterior thorax. According to the rule of nines, the nurse would assess that this injury constitutes which body percentage? Fill in the blank.

54%

The nurse is caring for a 71-kg patient during the first 12 hours after a thermal burn injury. Which outcomes indicate adequate fluid resuscitation? (Select all that apply.) a. Urine output is 46 mL/hr. b. Heart rate is 94 beats/min. c. Urine specific gravity is 1.040. d. Mean arterial pressure is 54 mm Hg. e. Systolic blood pressure is 88 mm Hg.

A, B a. Urine output is 46 mL/hr. b. Heart rate is 94 beats/min. Assessment of the adequacy of fluid resuscitation is best made using either urine output or cardiac factors. Urine output should be 0.5 to 1 mL/kg/hr (or 75 to 100 mL/hr for an electrical burn patient with evidence of hemoglobinuria/myoglobinuria). Cardiac factors include a mean arterial pressure (MAP) greater than 65 mm Hg, systolic BP greater than 90 mm Hg, and heart rate less than 120 beats/min. Normal range for urine specific gravity is 1.003 to 1.030.

A patient is in acute respiratory distress syndrome (ARDS) from sepsis. Which measure would be implemented to maintain cardiac output? a. Administer IV crystalloid fluids. b. Place the patient on a strict fluid restriction. c. Position the patient in Trendelenburg position. d. Perform chest physiotherapy and assist with staged coughing.

a. Administer IV crystalloid fluids. Low cardiac output may necessitate crystalloid fluids in addition to lowering positive end-expiratory pressure (PEEP) or giving inotropes. The Trendelenburg position is not recommended to treat hypotension. Chest physiotherapy is unlikely to relieve decreased cardiac output. Fluid restriction would be an inappropriate intervention.

A nurse is caring for a patient with right lower lobe pneumonia who is obese. Which position will provide the best gas exchange? a. On the left side b. On the right side c. In the tripod position d. In the high-Fowler's position

a. On the left side The patient should be positioned with the "good" lung in the dependent position to improve the match between ventilation and perfusion. The obese patient's abdomen will limit respiratory excursion when sitting in the high-Fowler's or tripod positions.

A young adult patient who is in the rehabilitation phase 6 months after a severe face and neck burn tells the nurse, "I'm sorry that I'm still alive. My life will never be normal again." Which response should the nurse make? a. "Most people recover after a burn and feel satisfied with their lives." b. "It's true that your life may be different. What concerns you the most?" c. "Why do you feel that way? It will get better as your recovery progresses." d. "It is really too early to know how much your life will be changed by the burn."

b. "It's true that your life may be different. What concerns you the most?" This response acknowledges the patient's feelings and asks for more assessment data that will help in developing an appropriate plan of care to assist the patient with the emotional response to the burn injury. The other statements are accurate but do not acknowledge the anxiety and depression that the patient is expressing.

Which statement by the nurse when explaining the purpose of positive end-expiratory pressure (PEEP) to the patient's caregiver is accurate? a. "PEEP will push more air into the lungs during inhalation." b. "PEEP prevents the lung air sacs from collapsing during exhalation." c. "PEEP will prevent lung damage while the patient is on the ventilator." d. "PEEP allows the breathing machine to deliver 100% O2 to the lungs."

b. "PEEP prevents the lung air sacs from collapsing during exhalation." By preventing alveolar collapse during expiration, PEEP improves gas exchange and oxygenation. PEEP will not prevent lung damage (e.g., fibrotic changes that occur with ARDS), push more air into the lungs, or change the fraction of inspired oxygen (FIO2) delivered to the patient.

A patient with septic shock has a BP of 70/46 mm Hg, pulse of 136 beats/min, respirations of 32 breaths/min, temperature of 104° F, and blood glucose of 246 mg/dL. Which intervention ordered by the health care provider should the nurse implement first? a. Acetaminophen (Tylenol) 650 mg rectally. b. Administer normal saline IV at 500 mL/hr. c. Start norepinephrine to keep blood pressure above 90 mm Hg. d. Start insulin drip to maintain blood glucose at 110 to 150 mg/dL.

b. Administer normal saline IV at 500 mL/hr. Because of the decreased preload associated with septic shock, fluid resuscitation is the initial therapy. The other actions also are appropriate and should be initiated quickly as well.

When caring for a critically ill patient who is being mechanically ventilated, the nurse will monitor for which manifestation of multiple organ dysfunction syndrome (MODS)? a. Increased serum albumin b. Decreased respiratory compliance c. Increased gastrointestinal (GI) motility d. Decreased blood urea nitrogen (BUN)/creatinine ratio

b. Decreased respiratory compliance Manifestations of MODS include symptoms of respiratory distress, signs and symptoms of decreased renal perfusion, decreased serum albumin and prealbumin, decreased GI motility, acute neurologic changes, myocardial dysfunction, disseminated intravascular coagulation (DIC), and changes in glucose metabolism

The nurse would recognize which assessment finding as suggestive of sepsis? a. Sudden diuresis unrelated to drug therapy b. Hyperglycemia in the absence of diabetes c. Respiratory rate of seven breaths per minute d. Bradycardia with sudden increase in blood pressure

b. Hyperglycemia in the absence of diabetes Hyperglycemia in patients with no history of diabetes is a diagnostic criterion for sepsis. Oliguria, not diuresis, typically accompanies sepsis along with tachypnea and tachycardia.

Which finding by the nurse should result in postponing the spontaneous breathing trial for a patient receiving positive pressure ventilation? a. Enteral nutrition is being given through an orogastric tube. b. Scattered rhonchi are heard when auscultating breath sounds. c. New ST segment elevation is observed on the cardiac monitor. d. Hydromorphone (Dilaudid) is being used to treat postoperative pain

c. New ST segment elevation is observed on the cardiac monitor. Myocardial ischemia is a contraindication for ventilator weaning. The ST segment elevation is an indication that weaning should be postponed until further investigation and/or treatment for myocardial ischemia can be done. Ventilator weaning can proceed when opioids are used for pain management, abnormal lung sounds are present, or enteral nutrition is being delivered.

A patient who was found unconscious in a burning house is brought to the emergency department by ambulance. The nurse notes that the patient's skin color is bright red. Which action should the nurse take first? a. Insert two large-bore IV lines. b. Check the patient's orientation. c. Place the patient on 100% O2 using a nonrebreather mask. d. Assess for singed nasal hair and dark oral mucous membranes.

c. Place the patient on 100% O2 using a nonrebreather mask. The patient's history and skin color suggest carbon monoxide poisoning, which should be treated by rapidly starting O2 at 100%. The other actions can be taken after the action to correct gas exchange

The nurse is reviewing laboratory results on a patient who had a large burn 48 hours ago. Which result requires priority action by the nurse? a. Hematocrit of 53% b. Serum sodium of 147 mEq/L c. Serum potassium of 6.1 mEq/L d. Blood urea nitrogen of 37 mg/dL

c. Serum potassium of 6.1 mEq/L Hyperkalemia can lead to life-threatening dysrhythmias. The patient needs cardiac monitoring and immediate treatment to lower the potassium level. The other laboratory values are also abnormal and require changes in treatment, but they are not as immediately life threatening as the elevated potassium level.

Which nursing action is a priority for a patient who has suffered a burn injury while working on an electrical power line? a. Inspect the contact burns. b. Check the blood pressure. c. Stabilize the cervical spine. d. Assess alertness and orientation.

c. Stabilize the cervical spine. Cervical spine injuries are often associated with electrical burns. Therefore, stabilization of the cervical spine takes precedence after airway management. The other actions are also included in the emergent care after electrical burns, but the most important action is to avoid spinal cord injury.

Esomeprazole is prescribed for a patient who incurred extensive burn injuries 5 days ago. Which nursing assessment would best evaluate the effectiveness of the drug? a. Bowel sounds b. Stool frequency c. Stool occult blood d. Abdominal distention

c. Stool occult blood H2 blockers and proton pump inhibitors are given to prevent Curling's ulcer in the patient who has sustained burn injuries. Proton pump inhibitors usually do not affect bowel sounds, stool frequency, or appetite.

During change-of-shift report on a medical unit, the nurse learns that a patient with aspiration pneumonia who was admitted with respiratory distress has become increasingly agitated. Which action should the nurse take first? a. Give the prescribed PRN sedative drug. b. Offer reassurance and reorient the patient. c. Use pulse oximetry to check the oxygen saturation. d. Notify the health care provider about the patient's status.

c. Use pulse oximetry to check the oxygen saturation. Agitation may be an early indicator of hypoxemia. The other actions may also be appropriate, depending on the findings about O2 saturation.

The nurse responding to a ventilator alarm finds the patient lying in bed gasping and the endotracheal tube on the floor. Which action should the nurse take next? a. Activate the rapid response team. b. Provide reassurance to the patient. c. Call the health care provider to reinsert the tube. d. Manually ventilate the patient with 100% oxygen.

d. Manually ventilate the patient with 100% oxygen. The nurse should ensure maximal patient oxygenation by manually ventilating with a bag-valve-mask system. Offering reassurance to the patient, notifying the health care provider about the need to reinsert the tube, and activating the rapid response team are also appropriate after the nurse has stabilized the patient's oxygenation.

A client taking calcium carbonate chewable tablets and ranitidine is on nothing by mouth (NPO) status and has a nasogastric (NG) tube in place after suffering bilateral burns to the legs. The nurse determines that the client's gastrointestinal (GI) status is least satisfactory if which finding is noted on assessment? 1. Gastric pH of 3 2. Absence of abdominal discomfort 3. GI drainage that is guaiac negative 4. Presence of hypoactive bowel sounds

1. Gastric pH of 3

A burn-injured client is receiving treatments of topical mafenide acetate to the site of injury. The nurse should monitor the client for which systemic effect that can occur from the use of this medication? 1.Acidosis 2.Alkalosis 3.Hypotension 4.Hypertension

1.Acidosis

The nurse is reviewing the laboratory test results for a client admitted to the burn unit 3 hours after an explosion that occurred at a worksite. The client has a severe burn injury that covers 35% of the total body surface area (TBSA). The nurse is most likely to note which finding on the laboratory report? 1.Hematocrit 60% (0.60) 2.Serum albumin 4.8 g/dL (48 g/L) 3.Serum sodium 144 mEq/L (144 mmol/L) 4.White blood cell (WBC) count 9000 mm3 (9 × 109/L)

1.Hematocrit 60% (0.60)

The nurse is developing a plan of care for a client at risk for acute respiratory distress syndrome (ARDS). As part of the plan, the nurse will assess for which sign or symptom for early detection of this disorder? 1.Edema 2.Dyspnea 3.Frothy sputum 4.Diminished breath sounds

2.Dyspnea

The nurse is evaluating fluid resuscitation attempts in the burn client. Which finding indicates adequate fluid resuscitation? 1.Disorientation to time only 2.Heart rate of 95 beats/minute 3.+1 palpable peripheral pulses 4.Urine output of 30 mL over the past 2 hours

2.Heart rate of 95 beats/minute

The postanesthesia care unit (PACU) has several patients with endotracheal tubes. Which patient should receive the least amount of endotracheal suctioning? a. Transplantation of a kidney b. Replacement of aortic valve c. Cerebral aneurysm resection d. Formation of an ileal conduit

c. Cerebral aneurysm resection The nurse should avoid suctioning the patient after a craniotomy until it is necessary because suctioning will increase this patient's intracranial pressure. The patients with a kidney transplantation, aortic valve replacement, or formation of an ileal conduit will not be negatively affected by suctioning, although it should only be done when needed, not routinely.

The charge nurse observes the following actions being taken by a new nurse on the burn unit. Which action by the new nurse would require immediate intervention by the charge nurse? a. The new nurse uses clean gloves when applying antibacterial cream to a burn wound. b. The new nurse obtains burn cultures when the patient has a temperature of 95.2° F (35.1° C). c. The new nurse gives PRN fentanyl (Sublimaze) IV to a patient 5 minutes before a dressing change. d. The new nurse calls the health care provider when a nondiabetic patient's serum glucose is elevated.

a. The new nurse uses clean gloves when applying antibacterial cream to a burn wound. Sterile gloves should be worn when applying medications or dressings to a burn. Hypothermia is an indicator of possible sepsis, and cultures are appropriate. Nondiabetic patients may need insulin because stress and high-calorie intake may lead to temporary hyperglycemia. Fentanyl peaks 5 minutes after IV administration and should be used just before and during dressing changes for pain management.

When admitting a patient with possible respiratory failure and a high PaCO2, which assessment information should be immediately reported to the health care provider? a. The patient appears somnolent. b. The patient reports feeling weak. c. The patient's blood pressure is 164/98. d. The patient's oxygen saturation is 90%.

a. The patient appears somnolent. Increasing somnolence will decrease the patient's respiratory rate and further increase the PaCO2 and respiratory failure. Rapid action is needed to prevent respiratory arrest. An SpO2 of 90%, weakness, and elevated blood pressure all require ongoing monitoring but are not indicators of possible impending respiratory arrest.

Prone positioning is being used for a patient with acute respiratory distress syndrome (ARDS). Which information obtained by the nurse indicates that the positioning is effective? a. The patient's PaO2 is 89 mm Hg, and the SaO2 is 91%. b. Endotracheal suctioning results in clear mucous return. c. Sputum and blood cultures show no growth after 48 hours. d. The skin on the patient's back is intact and without redness.

a. The patient's PaO2 is 89 mm Hg, and the SaO2 is 91%. The purpose of prone positioning is to improve the patient's oxygenation as indicated by the PaO2 and SaO2. The other information will be collected but does not indicate whether prone positioning has been effective.

The community health nurse is providing a teaching session to firefighters in a small community regarding care of a burn victim at the scene of injury. The nurse instructs the firefighters that in the event of a tar burn, which is the immediate action? 1.Cooling the injury with water 2.Removing all clothing immediately 3.Removing the tar from the burn injury 4.Leaving any clothing that is saturated with tar in place

1.Cooling the injury with water

The nurse is supervising a nursing student who is delivering care to a client with a burn injury to the chest. Nitrofurazone is prescribed to be applied to the site of injury. The nurse should intervene if the student planned to implement which action to apply the medication? 1.Wash the burn site. 2.Apply the medication with a sterile gloved hand. 3.Apply saline-soaked dressings over the medication. 4.Apply 1/16-inch (1.5-mm) film directly to the burn sites.

3.Apply saline-soaked dressings over the medication.

A client's arterial blood gas results reveal a PaO2 of 55 mm Hg. The client's admitting diagnosis is acute respiratory failure secondary to community-acquired pneumonia. What is the nurse's best action? 1.Repeat arterial blood gas testing. 2.Maintain continuous pulse oximetry. 3.Notify the primary health care provider (PHCP). 4.Decrease the amount of oxygen administered.

3.Notify the primary health care provider (PHCP).

A client with no history of respiratory disease is admitted to the hospital with respiratory failure. Which results on the arterial blood gas report that are consistent with this disorder should the nurse expect to note? 1.PaO2 58 mm Hg, PaCO2 32 mm Hg 2.PaO2 60 mm Hg, PaCO2 45 mm Hg 3.PaO2 49 mm Hg, PaCO2 52 mm Hg 4.PaO2 73 mm Hg, PaCO2 62 mm Hg

3.PaO2 49 mm Hg, PaCO2 52 mm Hg

When assessing a patient who spilled hot oil on the right leg and foot, the nurse notes dry, pale, and hard skin. The patient states that the burn is not painful. What term would the nurse use to document the burn depth? a. First-degree skin destruction b. Full-thickness skin destruction c. Deep partial-thickness skin destruction d. Superficial partial-thickness skin destruction

b. Full-thickness skin destruction With full-thickness skin destruction, the appearance is pale and dry or leathery, and the area is painless because of the associated nerve destruction. Erythema, swelling, and blisters point to a deep partial-thickness burn. With superficial partial-thickness burns, the area is red, but no blisters are present. First-degree burns exhibit erythema, blanching, and pain.

An older adult is moving into an independent living facility. What teaching will prevent this patient from being accidently burned in the new home? a. Encourage her to stop smoking. b. Install tap water anti-scald devices. c. Ensure all meals are cooked for her. d. Be sure she uses an open space heater.

b. Install tap water anti-scald devices. Installing tap water anti-scald devices will help prevent accidental scald burns that more easily occur in older people as their skin becomes drier and the dermis thinner. Cooking for her may be needed at times of illness or in the future, but she is moving to an independent living facility, so at this time she should not need this assistance. Stopping her from smoking may be helpful to prevent burns but may not be possible without the requirement by the facility. Using an open space heater would increase her risk of being burned and would not be encouraged.

A patient with circumferential burns of both legs develops a decrease in dorsalis pedis pulse strength and numbness in the toes. Which action should the nurse take first? a. Monitor the pulses every hour. b. Notify the health care provider. c. Elevate both legs above heart level with pillows. d. Encourage the patient to flex and extend the toes.

b. Notify the health care provider. The decrease in pulse and numbness in a patient with circumferential burns shows decreased circulation to the legs and the need for an escharotomy. Monitoring the pulses is not an adequate response to the decrease in circulation. Elevating the legs or increasing toe movement will not improve the patient's circulation.

When caring for a patient with acute respiratory distress syndrome (ARDS), which finding indicates therapy is appropriate? a. Arterial pH is 7.32. b. PaO2 is greater than or equal to 60 mm Hg. c. PEEP increased to 20 cm H2O caused BP to fall to 80/40. d. No change in PaO2 when patient is turned from supine to prone position.

b. PaO2 is greater than or equal to 60 mm Hg. The overall goal in caring for the patient with ARDS is for the PaO2 to be greater than or equal to 60 mm Hg with adequate lung ventilation to maintain a normal pH of 7.35 to 7.45. PEEP is usually increased for ARDS patients, but a dramatic reduction in BP indicates a complication of decreased cardiac output. A positive occurrence is a marked improvement in PaO2 from perfusion better matching ventilation when the anterior air-filled, nonatelectatic alveoli become dependent in the prone position.

ANSWER b. Albumin level and weight loss The patient's recent weight loss and low protein stores indicate possible muscle weakness, which make it more difficult for an older patient to recover from the effects of general anesthesia and immobility associated with the hip surgery. The other information will also be noted by the nurse but does not place the patient at higher risk for respiratory failure.

QUESTION The nurse reviews the electronic health record for a patient scheduled for a total hip replacement. Which assessment data shown in the accompanying figure increase the patient's risk for respiratory complications after surgery? a. Older age and anemia b. Albumin level and weight loss c. Recent arthroscopic procedure d. Confusion and disorientation to time

The nurse is caring for a patient who sustained a deep partial-thickness burn to the anterior chest area during a workplace accident 6 hours ago. Which assessment findings would the nurse identify as congruent with this type of burn? a. Skin is hard with a dry, waxy white appearance. b. Skin is shiny and red with clear, fluid-filled blisters. c. Skin is red and blanches when slight pressure is applied. d. Skin is leathery with visible muscles, tendons, and bones.

b. Skin is shiny and red with clear, fluid-filled blisters. Deep partial-thickness burns have fluid-filled vesicles that are red and shiny. They may appear wet (if vesicles have ruptured), and mild to moderate edema may be present. Superficial partial-thickness burns are red and blanch with pressure vesicles that appear 24 hours after the burn injury. Full-thickness burns are dry, waxy white, leathery, or hard, and there may be involvement of muscles, tendons, and bones.

Which nursing action prevents cross contamination when the patient's full-thickness burn wounds to the face are exposed? a. Using sterile gloves when removing dressings. b. Keeping the room temperature at 70° F (20° C). c. Wearing gown, cap, mask, and gloves during care. d. Giving IV antibiotics to prevent bacterial colonization.

c. Wearing gown, cap, mask, and gloves during care. Use of gowns, caps, masks, and gloves during all patient care will decrease the possibility of wound contamination for a patient whose burns are not covered. When removing contaminated dressings and washing the dirty wound, use nonsterile, disposable gloves. The room temperature should be kept at 85° F for patients with open burn wounds to prevent shivering. Systemic antibiotics are not well absorbed into deep burns because of the lack of circulation.

Which prescribed drug is best for the nurse to give before scheduled wound debridement on a patient with partial-thickness burns? a. ketorolac b. lorazepam (Ativan) c. gabapentin (Neurontin) d. hydromorphone (Dilaudid)

d. hydromorphone (Dilaudid) Opioid pain medications are the best choice for pain control. The other drugs are used as adjuvants to enhance the effects of opioids.

An adult client with a burn injury just arrived at the emergency department. Place the nursing actions in the care of this client in order of priority. All options must be used. 1.Assess for airway patency. 2.Obtain vital signs. 3.Administer oxygen as prescribed. 4.Elevate the extremities. 5.Keep the client warm. 6.Initiate an intravenous (IV) line and begin fluid replacement as prescribed.

1.Assess for airway patency. 3.Administer oxygen as prescribed. 2.Obtain vital signs. 6.Initiate an intravenous (IV) line and begin fluid replacement as prescribed. 4.Elevate the extremities. 5.Keep the client warm.

The industrial nurse is providing instructions to a group of employees regarding care to a client in the event of a chemical burn injury. The nurse instructs the employees that which is the first consideration in immediate care? 1.Removing all clothing, including gloves, shoes, and any undergarments 2.Determining the antidote for the chemical and placing the antidote on the burn site 3.Leaving all clothing in place until the client is brought to the emergency department 4.Lavaging the skin with water and avoiding brushing powdered chemicals off the clothing

1.Removing all clothing, including gloves, shoes, and any undergarments

The nurse is caring for a client with acute respiratory distress syndrome (ARDS). What should the nurse expect to note in the client? 1.Pallor 2.Low arterial PaO2 3.Elevated arterial PaO2 4.Decreased respiratory rate

2.Low arterial PaO2

A client at risk for shock secondary to pneumonia develops restlessness and is agitated and confused. Urinary output has decreased and the blood pressure is 92/68 mm Hg. The nurse minimally suspects which stage of shock based on this data? 1.Stage 1 2.Stage 2 3.Stage 3 4.Stage 4

2.Stage 2

An adult client trapped in a burning house has suffered burns to the back of the head, the upper half of the posterior trunk, and the back of both arms. Using the rule of nines, what percentage does the nurse determine the extent of the burn injury to be? Fill in the blank.

22.5%

The nurse estimates the extent of a burn using the rule of nines for a patient who has been admitted with deep partial-thickness burns of the anterior trunk and the entire left arm. What percentage of the patient's total body surface area (TBSA) has been injured?

27% When using the rule of nines, the anterior trunk is considered to cover 18% of the patient's body and the anterior (4.5%) and posterior (4.5%) left arm equals 9%.

The nurse is providing care for a client who sustained burns over 30% of the body from a fire. On assessment, the nurse notes that the client is edematous in both burned and unburned body areas. The client's wife asks why her husband "looks so swollen." What is the nurse's best response? 1."Constricted blood vessels have caused a loss of protein in the blood." 2."Leaking blood vessels have led to increased protein amounts in the blood." 3."Leaking blood vessels have led to decreased protein amounts in the blood." 4."Constricted blood vessels have led to increased protein amounts in the blood."

3."Leaking blood vessels have led to decreased protein amounts in the blood."

The nurse is performing an assessment on a client who sustained circumferential burns of both legs. Which assessment would be the initial priority in caring for this client? 1.Assessing heart rate 2.Assessing respiratory rate 3.Assessing peripheral pulses 4.Assessing blood pressure (BP)

3.Assessing peripheral pulses

The nurse is performing an assessment on a client admitted to the nursing unit who has sustained an extensive burn injury involving 45% of total body surface area. When planning for fluid resuscitation, the nurse should consider that fluid shifting to the interstitial spaces is greatest during which time period? 1.Immediately after the injury 2.Within 12 hours after the injury 3.Between 18 and 24 hours after the injury 4.Between 42 and 72 hours after the injury

3.Between 18 and 24 hours after the injury

The nurse is caring for a terminally ill client who is experiencing Cheyne-Stokes respirations. Which assessment finding should the nurse expect to note? 1.Continuous rapid regular breathing 2.Periods of apnea followed by bradypnea 3.Periods of apnea followed by deep rapid breathing 4.Periods of bradypnea followed by periods of tachypnea

3.Periods of apnea followed by deep rapid breathing

A burn client has been having 1% silver sulfadiazine applied to burns twice a day for the past 3 days. Which laboratory abnormality indicates that the client is experiencing a side or adverse effect of this medication? 1.Serum sodium of 120 mEq/L (120 mmol/L) 2.Serum potassium of 3.0 mEq/L (3.0 mmol/L) 3.White blood cell count of 3000 mm3 (3 × 109/L) 4.pH of 7.30, PaCO2 of 32 mm Hg (32 mmHg), HCO3- of 19 mEq/L (19 mmol/L)

3.White blood cell count of 3000 mm3 (3 × 109/L)

Silver sulfadiazine is prescribed for a client with a partial-thickness burn, and the nurse provides teaching about the medication. Which statement made by the client indicates a need for further teaching about the treatments? 1. "The medication is an antibacterial." 2. "The medication will help heal the burn." 3. "The medication should be applied directly to the wound." 4. "The medication is likely to cause stinging every time it is applied."

4. "The medication is likely to cause stinging every time it is applied."

Collagenase is prescribed for a client with a severe burn to the hand. The home care nurse provides instructions to the client regarding the use of the medication. Which client statement indicates an accurate understanding of the use of this medication? 1."I will apply the ointment at bedtime and in the morning." 2."I will apply the ointment once a day and leave it open to the air." 3."I will apply the ointment twice a day and leave it open to the air." 4."I will apply the ointment once a day and cover it with a sterile dressing."

4."I will apply the ointment once a day and cover it with a sterile dressing."

The nurse is caring for a client who sustained superficial partial-thickness burns on the anterior lower legs and anterior thorax. Which finding does the nurse expect to note during the resuscitation/emergent phase of the burn injury? 1.Decreased heart rate 2.Increased urinary output 3.Increased blood pressure 4.Elevated hematocrit levels

4.Elevated hematocrit levels

An 80-kg patient with burns over 30% of total body surface area (TBSA) is admitted to the burn unit. Using the Parkland formula of 4 mL/kg/%TBSA, what is the IV infusion rate (mL/hour) for lactated Ringer's solution that the nurse will give during the first 8 hours?

600 mL/hr The Parkland formula states that patients should receive 4 mL/kg/%TBSA burned during the first 24 hours. Half of the total volume is given in the first 8 hours and then the remaining half is given over 16 hours: 4 80 30 = 9600 mL total volume; 9600/2 = 4800 mL in the first 8 hours; 4800 mL/8 hr = 600 mL/hr.

Which preventive actions by the nurse will help limit the development of systemic inflammatory response syndrome (SIRS) in patients admitted to the hospital? (Select all that apply.) a. Ambulate postoperative patients as soon as possible after surgery. b. Use aseptic technique when manipulating invasive lines or devices. c. Remove indwelling urinary catheters as soon as possible after surgery. d. Administer prescribed antibiotics within 1 hour for patients with possible sepsis. e. Advocate for parenteral nutrition for patients who cannot take in adequate calories.

A, B, C, D a. Ambulate postoperative patients as soon as possible after surgery. b. Use aseptic technique when manipulating invasive lines or devices. c. Remove indwelling urinary catheters as soon as possible after surgery. d. Administer prescribed antibiotics within 1 hour for patients with possible sepsis. Because sepsis is the most frequent etiology for SIRS, measures to avoid infection such as removing indwelling urinary catheters as soon as possible, use of aseptic technique, and early ambulation should be included in the plan of care. Adequate nutrition is important in preventing SIRS. Enteral, rather than parenteral, nutrition is preferred when patients are unable to take oral feedings because enteral nutrition helps maintain the integrity of the intestine, thus decreasing infection risk. Antibiotics should be given within 1 hour after being prescribed to decrease the risk of sepsis progressing to SIRS.

The nurse is planning care for a patient with partial- and full-thickness skin destruction related to burn injury of the lower extremities. Which interventions will the nurse include in this patient's care? (Select all that apply.) a. Escharotomy b. Administration of diuretics c. IV and oral pain medications d. Daily cleansing and debridement e. Application of topical antimicrobial agent

A, C, D, E a. Escharotomy c. IV and oral pain medications d. Daily cleansing and debridement e. Application of topical antimicrobial agent An escharotomy (a scalpel incision through full-thickness eschar) is frequently required to restore circulation to compromised extremities. Daily cleansing and debridement as well as application of an antimicrobial ointment are expected interventions used to minimize infection and enhance wound healing. Pain control is essential in the care of a patient with a burn injury. With full-thickness burns, myoglobin and hemoglobin released into the bloodstream can occlude renal tubules. Adequate fluid replacement is used to prevent this occlusion.

A patient is admitted to the emergency department with first- and second-degree burns after being involved in a house fire. Which assessment findings would alert the nurse to the presence of an inhalation injury? (Select all that apply.) a. Singed nasal hair b. Generalized pallor c. Painful swallowing d. Burns on the upper extremities e. History of being involved in a large fire

A, C, E a. Singed nasal hair c. Painful swallowing e. History of being involved in a large fire Reliable clues to the occurrence of inhalation injury is the presence of facial burns, singed nasal hair, hoarseness, painful swallowing, darkened oral and nasal membranes, carbonaceous sputum, history of being burned in an enclosed space, altered mental status, and dyspnea.

When teaching the patient in the rehabilitation phase of a severe burn about performing range of motion (ROM), what explanations should the nurse give to the patient? (Select all that apply.) a. The exercises are the only way to prevent contractures. b. Active and passive ROM maintains function of body parts. c. ROM will reassure the patient that movement is still possible. d. Movement promotes mobilization of interstitial fluid back into the vascular bed. e. Active and passive ROM can only be done while the dressings are being changed.

B, C, D b. Active and passive ROM maintains function of body parts. c. ROM will reassure the patient that movement is still possible. d. Movement promotes mobilization of interstitial fluid back into the vascular bed. Active and passive ROM maintains function of body parts and reassures the patient that movement is still possible are the explanations that should be used. Contractures are prevented with ROM and splints. Movement facilitates mobilization of fluid in interstitial fluid back into the vascular bed. Although it is good to collaborate with physical therapy to perform ROM during dressing changes because the patient has already taken analgesics, ROM can and should be done throughout the day.

Which actions should the nurse start to reduce the risk for ventilator-associated pneumonia (VAP)? (Select all that apply.) a. Obtain arterial blood gases daily. b. Provide a "sedation holiday" daily. c. Give prescribed pantoprazole (Protonix). d. Elevate the head of the bed to at least 30 degrees. e. Provide oral care daily with chlorhexidine (0.12%) solution.

B,C,D,E b. Provide a "sedation holiday" daily. c. Give prescribed pantoprazole (Protonix). d. Elevate the head of the bed to at least 30 degrees. e. Provide oral care daily with chlorhexidine (0.12%) solution. All these interventions are part of the ventilator bundle that is recommended to prevent VAP. Arterial blood gases may be done daily but are not always necessary and do not help prevent VAP.

Which interventions should the nurse perform before using an open-suctioning technique for a patient with an endotracheal (ET) tube? (Select all that apply.) a. Put on clean gloves. b. Administer a bronchodilator. c. Perform a cardiopulmonary assessment. d. Hyperoxygenate the patient for 30 seconds. e. Perform hand hygiene before performing the procedure. f. Insert a few drops of normal saline into the ET to break up secretions.

C, D, E c. Perform a cardiopulmonary assessment. d. Hyperoxygenate the patient for 30 seconds. e. Perform hand hygiene before performing the procedure. Suctioning is preceded by a thorough assessment and hyperoxygenation for 30 seconds. Sterile, not clean, gloves are necessary, and it is not necessary to give a bronchodilator. Instillation of normal saline into the ET tube is not an accepted standard practice.

The nurse in collaboration with respiratory therapy is determining a patient's readiness to wean from the ventilator. Which findings indicate the patient is not a candidate for weaning? (Select all that apply.) a. Minute volume of 8 L/min b. Patient follows commands c. Serum hemoglobin of 6 g/dL d. Respirations of 28 breaths/min e. Mean arterial pressure (MAP) of 45 mmHg f. Negative inspiratory force (NIF) of -15 cm H2O

C, E, F c. Serum hemoglobin of 6 g/dL e. Mean arterial pressure (MAP) of 45 mmHg f. Negative inspiratory force (NIF) of -15 cm H2O Findings that support readiness for weaning are minute volume of 8 L/min, patient is alert and follow commands, and respirations of 28 breaths/min. Findings that indicate the patient is not ready for weaning include serum hemoglobin of 6 g/dL, mean arterial pressure (MAP) of 45 mm Hg, and negative inspiratory force (NIF) of −15 cm H2O. Extubating a patient with severe anemia, poor perfusion, and weakened breathing effort will likely result in poor outcomes such as worsening of condition and reintubation.

In which order will the nurse take these actions when doing a dressing change for a partial-thickness burn wound on a patient's chest? (Put a comma and a space between each answer choice [A, B, C, D, E].) a. Apply sterile gauze dressing. b. Document wound appearance. c. Apply silver sulfadiazine cream. d. Give IV fentanyl (Sublimaze). e. Clean wound with saline-soaked gauze.

D, E, C, A, B d. Give IV fentanyl (Sublimaze). e. Clean wound with saline-soaked gauze. c. Apply silver sulfadiazine cream. a. Apply sterile gauze dressing. b. Document wound appearance. Because partial-thickness burns are very painful, the nurse's first action should be to give pain medications. The wound will then be cleaned, antibacterial cream applied, and covered with a new sterile dressing. The last action should be to document the appearance of the wound.

In which order will the nurse take these actions when assisting with oral intubation of a patient in respiratory distress? (Put a comma and a space between each answer choice [A, B, C, D, E].) a. Obtain a portable chest-x-ray. b. Position the patient in the supine position. c. Inflate the cuff of the endotracheal tube after insertion. d. Attach an end-tidal CO2 detector to the endotracheal tube. e. Oxygenate the patient with a bag-valve-mask device for several minutes.

E, B, C, D, A e. Oxygenate the patient with a bag-valve-mask device for several minutes. b. Position the patient in the supine position. c. Inflate the cuff of the endotracheal tube after insertion. d. Attach an end-tidal CO2 detector to the endotracheal tube. a. Obtain a portable chest-x-ray. The patient is preoxygenated with a bag-valve-mask system for 3 to 5 minutes before intubation and then placed in a supine position. After the intubation, the cuff on the endotracheal tube is inflated to occlude and protect the airway. Tube placement is assessed first with an end-tidal CO2 sensor and then with chest x-ray examination.

The health care provider prescribes the following interventions for a 67-kg patient who has septic shock with a blood pressure of 70/42 mm Hg and O2 saturation of 90% on room air. In which order will the nurse implement the actions? (Put a comma and a space between each answer choice [A, B, C, D, E].) a. Obtain blood and urine cultures. b. Give vancomycin by IV infusion. c. Start norepinephrine 0.5 mcg/min. d. Infuse normal saline 2000 mL over 30 minutes. e. Administer oxygen to keep O2 saturation above 95%.

E, D, C, A, B e. Administer oxygen to keep O2 saturation above 95%. d. Infuse normal saline 2000 mL over 30 minutes. c. Start norepinephrine 0.5 mcg/min. a. Obtain blood and urine cultures. b. Give vancomycin by IV infusion. The initial action for this hypotensive and hypoxemic patient should be to improve the O2 saturation, followed by infusion of IV fluids and vasopressors to improve perfusion. Cultures should be obtained before giving antibiotics.

ANSWER c. Platelet count and presence of petechiae The low platelet count and presence of petechiae suggest that the patient may have disseminated intravascular coagulation and that multiple organ dysfunction syndrome is developing. The other information will be discussed with the health care provider but does not show that the patient's condition is deteriorating or that a change in therapy is needed immediately.

QUESTION After reviewing the information shown in the accompanying figure for a patient with pneumonia and sepsis, which information is most important to report to the health care provider? a. Temperature and IV site appearance b. Oxygen saturation and breath sounds c. Platelet count and presence of petechiae d. Blood pressure, pulse rate, respiratory rate

The nurse is providing emergent care for a patient with a possible inhalation injury sustained in a house fire. The patient is anxious and disoriented, and the skin is a cherry red color. What is the priority action by the nurse? a. Administer 100% humidified oxygen. b. Teach the patient deep breathing exercises. c. Encourage the patient to express his feelings. d. Assist the patient to a high Fowler's position.

a. Administer 100% humidified oxygen. Carbon monoxide (CO) poisoning may occur in house fires. CO displaces oxygen on the hemoglobin molecule resulting in hypoxia. High levels of CO in the blood result in a skin color that is described as cherry red. Hypoxia may cause anxious behaviors and altered mental status. Emergency treatment for inhalation injury and CO poisoning includes the immediate administration of 100% humidified oxygen. The other interventions are appropriate for inhalation injury but are not as urgent as oxygen administration.

A 78-kg patient in septic shock has a pulse rate of 120 beats/min with low central venous pressure and pulmonary artery wedge pressure. After initial fluid volume resuscitation, the patient's urine output has been 30 mL/hr for the past 3 hours. Which order by the health care provider should the nurse question? a. Administer furosemide (Lasix) 40 mg IV. b. Increase normal saline infusion to 250 mL/hr. c. Give hydrocortisone (Solu-Cortef) 100 mg IV. d. Use norepinephrine to keep systolic BP above 90 mm Hg.

a. Administer furosemide (Lasix) 40 mg IV. Furosemide will lower the filling pressures and renal perfusion further for the patient with septic shock. Patients in septic shock need large amounts of fluid replacement. If the patient is still hypotensive after initial volume resuscitation with minimally 30 mL/kg, vasopressors such as norepinephrine may be added. IV corticosteroids may be considered for patients in septic shock who cannot maintain an adequate BP with vasopressor therapy despite fluid resuscitation.

When caring for older adult patients with respiratory failure, the nurse will add which intervention to individualize care? a. Assess frequently for manifestations of delirium. b. Position the patient in the supine position primarily. c. Provide early endotracheal intubation to reduce complications. d. Delay activity and ambulation to provide additional healing time.

a. Assess frequently for manifestations of delirium. Older adult patients are more predisposed to delirium and health care-associated infections. Individualizing the older patient's care plan to address these factors will improve care. Older adult patients are not required to remain in a supine position only and should increase activity as soon as stability is determined. Endotracheal intubation is not provided early, and noninvasive positive pressure ventilation may be considered as an alternative. The nurse should consider that the aging process leads to decreased lung elastic recoil, weakened lung muscles and reduced gas exchange, which may make the patient difficult to wean from the ventilator.

The nurse is providing care for an older adult patient who has a low partial pressure of oxygen in arterial blood (PaO2) due to worsening left-sided pneumonia. Which intervention should the nurse use to help the patient mobilize his secretions? a. Augmented coughing or huff coughing b. Positioning the patient side-lying on his left side c. Frequent and aggressive nasopharyngeal suctioning d. Application of noninvasive positive pressure ventilation

a. Augmented coughing or huff coughing Augmented coughing and huff coughing techniques may aid the patient in the mobilization of secretions. If positioned side-lying, the patient should be positioned on his right side (good lung down) for improved perfusion and ventilation. Suctioning may be indicated but should always be performed cautiously because of the risk of hypoxia. NIPPV is inappropriate in the treatment of patients with excessive secretions.

A patient arrives in the emergency department with facial and chest burns caused by a house fire. Which action should the nurse take first? a. Auscultate for breath sounds. b. Determine the extent and depth of the burns. c. Give the prescribed hydromorphone (Dilaudid). d. Infuse the prescribed lactated Ringer's solution.

a. Auscultate for breath sounds. A patient with facial and chest burns is at risk for inhalation injury and assessment of airway and breathing is the priority. The other actions will be completed after airway management is assured.

A nurse is caring for a patient who is orally intubated and receiving mechanical ventilation. To decrease the risk for ventilator-associated pneumonia, which action will the nurse include in the plan of care? a. Elevate head of bed to 30 to 45 degrees. b. Give enteral feedings at no more than 10 mL/hr. c. Suction the endotracheal tube every 2 to 4 hours. d. Limit the use of positive end-expiratory pressure.

a. Elevate head of bed to 30 to 45 degrees. Elevation of the head decreases the risk for aspiration. Positive end-expiratory pressure is frequently needed to improve oxygenation in patients receiving mechanical ventilation. Suctioning should be done only when the patient assessment indicates that it is necessary. Enteral feedings should provide adequate calories for the patient's high energy needs.

A patient with acute respiratory distress syndrome (ARDS) and acute kidney injury has several drugs prescribed. Which drug should the nurse discuss with the health care provider before giving? a. Gentamicin 60 mg IV b. Pantoprazole (Protonix) 40 mg IV c. Sucralfate (Carafate) 1 gram per NG tube d. Methylprednisolone (Solu-Medrol) 60 mg IV

a. Gentamicin 60 mg IV Gentamicin, which is one of the aminoglycoside antibiotics, is potentially nephrotoxic, and the nurse should clarify the drug and dosage with the health care provider before administration. The other drugs are appropriate for the patient with ARDS.

A patient with acute respiratory distress syndrome (ARDS) is on positive pressure ventilation (PPV). The patient's cardiac index is 1.4 L/min and pulmonary artery wedge pressure is 8 mm Hg. What order by the provider would the nurse to question? a. Increase PEEP from 10 to 15 cm H2O. b. Start a dobutamine infusion at 3 mcg/kg/min. c. Give 1 unit of packed RBCs over the next 2 hours. d. Change the maintenance IV rate from 75 to 125 mL/hr.

a. Increase PEEP from 10 to 15 cm H2O. Patients on PPV and PEEP often have decreased cardiac output (CO) and cardiac index (CI). High levels of PEEP increase intrathoracic pressure and cause decreased venous return which results in decreased CO. Interventions to improve CO include lowering the PEEP, giving crystalloid fluids or colloid solutions, and use of inotropic drugs (e.g., dobutamine, dopamine). Packed red blood cells may also be administered to improve CO and oxygenation if the hemoglobin is less than 9 or 10 mg/dL.

When caring for a patient in acute septic shock, what should the nurse anticipate? a. Infusing large amounts of IV fluids b. Administering osmotic and/or loop diuretics c. Administering IV diphenhydramine (Benadryl) d. Assisting with insertion of a ventricular assist device (VAD)

a. Infusing large amounts of IV fluids Septic shock is characterized by a decreased circulating blood volume. Volume expansion with the administration of IV fluids is the cornerstone of therapy. Administering diuretics is inappropriate. VADs are useful for cardiogenic shock, not septic shock. Diphenhydramine may be used for anaphylactic shock but would not be helpful with septic shock.

The nurse is planning to change the dressing that covers a deep partial-thickness burn of the right lower leg. Which prescribed medication should the nurse administer to the patient 30 minutes before the scheduled dressing change? a. Morphine b. Sertraline c. Zolpidem d. Alprazolam

a. Morphine Deep partial-thickness burns result in severe pain related to nerve injury. The nurse should plan to administer analgesics before the dressing change to promote patient comfort. Morphine is a common opioid used for pain control. Sedative/hypnotics and antidepressant agents also can be given with analgesics to control the anxiety, insomnia, and depression that patients may have.

The nurse is caring for a patient who has developed acute respiratory failure. Which medication is used to decrease patient pulmonary congestion and agitation? a. Morphine b. Albuterol c. Azithromycin d. Methylprednisolone

a. Morphine For a patient with acute respiratory failure related to the heart, morphine is used to decrease pulmonary congestion as well as anxiety, agitation, and pain. Albuterol is used to reduce bronchospasm. Azithromycin is used for pulmonary infections. Methylprednisolone is used to reduce airway inflammation and edema.

A patient has been admitted with dehydration and hypotension after having vomiting and diarrhea for 4 days. Which finding is most important for the nurse to report to the health care provider? a. New onset of confusion b. Decreased bowel sounds c. Heart rate 112 beats/min d. Pale, cool, and dry extremities

a. New onset of confusion The changes in mental status are indicative that the patient is in the progressive stage of shock and that rapid intervention is needed to prevent further deterioration. The other information is consistent with compensatory shock.

Which information about a patient who is receiving cisatracurium (Nimbex) to prevent asynchronous breathing with the positive pressure ventilator requires action by the nurse? a. No sedative is ordered for the patient. b. The patient does not respond to voice. c. The patient's oxygen saturation is 90% to 93%. d. The patient has no cough reflex when suctioned.

a. No sedative is ordered for the patient. Because neuromuscular blockade is extremely anxiety provoking, it is essential that patients who are receiving neuromuscular blockade receive concurrent sedation and analgesia. Absence of response to stimuli is expected in patients receiving neuromuscular blockade. The O2 saturation is adequate.

A nurse is caring for a patient with acute respiratory distress syndrome (ARDS) who is receiving mechanical ventilation using synchronized intermittent mandatory ventilation (SIMV). The settings include fraction of inspired oxygen (FIO2) of 80%, tidal volume of 450, rate of 16/minute, and positive end-expiratory pressure (PEEP) of 5 cm. Which assessment finding is most important for the nurse to report to the health care provider? a. O2 saturation of 99% b. Heart rate 106 beats/min c. Crackles audible at lung bases d. Respiratory rate 22 breaths/min

a. O2 saturation of 99% The FIO2 of 80% increases the risk for O2 toxicity. Because the patient's O2 saturation is 99%, a decrease in FIO2 is needed to avoid toxicity. The other patient data would be typical for a patient with ARDS and would not be the most important data to report to the health care provider.

The nurse is caring for a patient with acute respiratory distress syndrome (ARDS) who is on pressure support ventilation (PSV), fraction of inspired oxygen (FIO2) at 80%, and positive end-expiratory pressure (PEEP) at 15 cm H2O. The patient weighs 72 kg. What finding would indicate that treatment is effective? a. PaO2 of 60 mm Hg b. Tidal volume of 700 mL c. Cardiac output of 2.7 L/min d. Inspiration to expiration ratio of 1:2

a. PaO2 of 60 mm Hg Severe hypoxemia (PaO2 less than 40 mm Hg) occurs with ARDS, and PEEP is increased to improve oxygenation and prevent oxygen toxicity by reducing FIO2. A PaO2 level of 60 mm Hg indicates that treatment is effective and oxygenation status has improved. Decreased cardiac output is a complication of PEEP. Normal cardiac output is 4 to 8 L/minute. Normal tidal volume is 6 to 10 mL/kg. PSV delivers a preset pressure, but the tidal volume varies with each breath. I:E ratio is usually set at 1:2 to 1:1.5 and does not indicate patient improvement.

The nurse is caring for a patient who is intubated and receiving positive pressure ventilation to treat acute respiratory distress syndrome (ARDS). Which finding is most important to report to the health care provider? a. Red-brown drainage from nasogastric tube b. Blood urea nitrogen (BUN) level 32 mg/dL c. Scattered coarse crackles heard throughout lungs d. Arterial blood gases: pH of 7.31, PaCO2 of 50, and PaO2 of 68

a. Red-brown drainage from nasogastric tube The nasogastric drainage indicates possible gastrointestinal bleeding or stress ulcer and should be reported. The pH and PaCO2 are slightly abnormal, but current guidelines advocating for permissive hypercapnia indicate that these would not indicate an immediate need for a change in therapy. The BUN is slightly elevated but does not indicate an immediate need for action. Adventitious breath sounds are often heard in patients with ARDS.

An employee spills industrial acid on both arms and legs at work. What action should the occupational health nurse take? a. Remove nonadherent clothing and wristwatch. b. Apply an alkaline solution to the affected area. c. Place a cool compress on the area of exposure. d. Cover the affected area with dry, sterile dressings.

a. Remove nonadherent clothing and wristwatch. With chemical burns, the first action is to remove the chemical from contact with the skin as quickly as possible. Remove nonadherent clothing, shoes, watches, jewelry, glasses, or contact lenses (if the face was exposed). Flush the chemical from the wound and surrounding area with copious amounts of saline solution or water. Covering the affected area or placing cool compresses on the area will leave the chemical in contact with the skin. Application of an alkaline solution can cause more injury.

The nurse is caring for a patient who has septic shock. Which assessment finding is most important for the nurse to report to the health care provider? a. Skin cool and clammy b. Heart rate of 118 beats/min c. Blood pressure of 92/56 mm Hg d. O2 saturation of 93% on room air

a. Skin cool and clammy Because patients in the early stage of septic shock have warm and dry skin, the patient's cool and clammy skin indicates that shock is progressing. The other information will also be reported but does not indicate deterioration of the patient's status.

A patient with a burn inhalation injury is receiving albuterol for the treatment of bronchospasm. What is the most important adverse effect of this medication for the nurse to monitor? a. Tachycardia b. Restlessness c. Hypokalemia d. Gastrointestinal (GI) distress

a. Tachycardia Albuterol stimulates β-adrenergic receptors in the lungs to cause bronchodilation. However, it is a noncardioselective agent, so it also stimulates the β-receptors in the heart to increase the heart rate. Restlessness and GI upset may occur but will decrease with use. Hypokalemia does not occur with albuterol.

A patient with respiratory failure has arterial pressure-based cardiac output (APCO) monitoring and is receiving mechanical ventilation with peak end-expiratory pressure (PEEP) of 12 cm H2O. Which information indicates that a change in the ventilator settings may be required? a. The arterial pressure is 90/46. b. The heart rate is 58 beats/min. c. The stroke volume is increased. d. The stroke volume variation is 12%.

a. The arterial pressure is 90/46. The hypotension suggests that the high intrathoracic pressure caused by the PEEP may be decreasing venous return and (potentially) cardiac output. The other assessment data would not be a direct result of PEEP and mechanical ventilation.

The nurse is caring for a patient who arrived in the emergency department with acute respiratory distress. Which assessment finding by the nurse requires the most rapid action? a. The patient's PaO2 is 45 mm Hg. b. The patient's PaCO2 is 33 mm Hg. c. The patient's respirations are shallow. d. The patient's respiratory rate is 32 breaths/min.

a. The patient's PaO2 is 45 mm Hg. The PaO2 indicates severe hypoxemia and respiratory failure. Rapid action is needed to prevent further deterioration of the patient. Although the shallow breathing, rapid respiratory rate, and low PaCO2 also need to be addressed, the most urgent problem is the patient's poor oxygenation.

A patient who is orally intubated and receiving mechanical ventilation is anxious and is "fighting" the ventilator. Which action should the nurse take next? a. Verbally coach the patient to breathe with the ventilator. b. Sedate the patient with the ordered PRN lorazepam (Ativan). c. Manually ventilate the patient with a bag-valve-mask device. d. Increase the rate for the ordered propofol (Diprivan) infusion.

a. Verbally coach the patient to breathe with the ventilator. The initial response by the nurse should be to try to decrease the patient's anxiety by coaching the patient about how to coordinate respirations with the ventilator. The other actions may also be helpful if the verbal coaching is ineffective in reducing the patient's anxiety.

A patient is admitted to the burn unit with second- and third-degree burns covering the face, entire right upper extremity, and right anterior trunk area. Using the rule of nines, what should the nurse calculate the extent of these burns as being? a. 18% b. 22.5% c. 27% d. 36%

b. 22.5% Using the rule of nines, for these second- and third-degree burns, the face encompasses 4.5% of the body area, the entire right arm encompasses 9% of the body area, and the entire anterior trunk encompasses 18% of the body area. Because the patient has burns on only the right side of the anterior trunk, the nurse would assess that burn as encompassing half of the 18%, or 9%. Therefore, adding the three areas together (4.5 + 9 + 9), the nurse would correctly calculate the extent of this patient's burns to cover about 22.5% of the total body surface area.

Which patient should the nurse assess first? a. A patient with burns who reports a level 8 (0 to 10 scale) pain. b. A patient with smoke inhalation who has wheezes and altered mental status. c. A patient with full-thickness leg burns who is scheduled for a dressing change. d. A patient with partial thickness burns who is receiving IV fluids at 500 mL/hr.

b. A patient with smoke inhalation who has wheezes and altered mental status. This patient has evidence of lower airway injury and hypoxemia and should be assessed at once to determine the need for O2 or intubation (or both). The other patients should be assessed as rapidly as possible, but they do not have evidence of life-threatening complications.

The oxygen saturation (SpO2) for a patient with left lower lobe pneumonia is 90%. The patient has wheezes and a weak cough effort. Which action should the nurse take? a. Position the patient on the left side. b. Assist the patient with staged coughing. c. Place a humidifier in the patient's room. d. Schedule a 4-hour rest period for the patient.

b. Assist the patient with staged coughing. The patient's assessment indicates that assisted coughing is needed to help remove secretions, which will improve oxygenation. A 4-hour rest period at this time may allow the O2 saturation to drop further. Humidification will not be helpful unless the secretions can be mobilized. Positioning on the left side may cause a further decrease in oxygen saturation because perfusion will be directed more toward the more poorly ventilated lung.

A patient admitted with burns over 30% of the body surface 2 days ago now has dramatically increased urine output. Which action should the nurse plan to support maintaining kidney function? a. Monitoring white blood cells (WBCs). b. Continuing to measure the urine output. c. Assessing that blisters and edema have subsided. d. Encouraging the patient to eat adequate calories.

b. Continuing to measure the urine output. The patient's urine output indicates that the patient is entering the acute phase of the burn injury and moving on from the emergent stage. At the end of the emergent phase, capillary permeability normalizes, and the patient begins to diurese large amounts of urine with a low specific gravity. Although this may occur at about 48 hours, it may be longer in some patients. Blisters and edema begin to resolve, but this process requires more time. WBCs may increase or decrease, based on the patient's immune status and any infectious processes. The WBC count does not indicate kidney function. Although adequate nutrition is important for healing, it does not ensure adequate kidney functioning.

A nurse is caring for a patient who has burns of the ears, head, neck, and right arm and hand. The nurse should place the patient in which position? a. Place the right arm and hand flexed in a position of comfort. b. Elevate the right arm and hand on pillows and extend the fingers. c. Assist the patient to a supine position with a small pillow under the head. d. Position the patient in a side-lying position with rolled towel under the neck.

b. Elevate the right arm and hand on pillows and extend the fingers. The right hand and arm should be elevated to reduce swelling and the fingers extended to avoid flexion contractures (even though this position may not be comfortable for the patient). The patient with burns of the ears should not use a pillow for the head because this will put pressure on the ears, and the pillow may stick to the ears. Patients with neck burns should not use a pillow or rolled towel because the head should be kept in an extended position to avoid contractures.

A patient with respiratory failure has a respiratory rate of 6 breaths/min and an oxygen saturation (SpO2) of 78%. The patient is increasingly lethargic. Which intervention will the nurse anticipate? a. Administration of 100% O2 by non-rebreather mask b. Endotracheal intubation and positive pressure ventilation c. Insertion of a mini-tracheostomy with frequent suctioning d. Initiation of continuous positive pressure ventilation (CPAP)

b. Endotracheal intubation and positive pressure ventilation The patient's lethargy, low respiratory rate, and SpO2 indicate the need for mechanical ventilation with ventilator-controlled respiratory rate. Giving high-flow O2 will not be helpful because the patient's respiratory rate is so low. Insertion of a mini-tracheostomy will promote removal of secretions, but it will not improve the patient's respiratory rate or oxygenation. CPAP requires that the patient initiate an adequate respiratory rate to allow adequate gas exchange.

While caring for a patient who has been admitted with a pulmonary embolism, the nurse notes a change in the patient's oxygen saturation (SpO2) from 94% to 88%. Which action should the nurse take? a. Suction the patient's oropharynx. b. Increase the prescribed O2 flowrate. c. Teach the patient to cough and deep breathe. d. Help the patient to sit in a more upright position.

b. Increase the prescribed O2 flowrate. Increasing O2 flowrate will usually improve O2 saturation in patients with ventilation-perfusion mismatch, as occurs with pulmonary embolism. Because the problem is with perfusion, actions that improve ventilation, such as deep breathing and coughing, sitting upright, and suctioning, are not likely to improve oxygenation.

A patient has just been admitted with a 40% total body surface area (TBSA) burn injury. To maintain adequate nutrition, the nurse should plan to take which action? a. Administer vitamins and minerals intravenously. b. Insert a feeding tube and initiate enteral nutrition. c. Infuse total parenteral nutrition via a central catheter. d. Encourage an oral intake of at least 5000 kcal per day.

b. Insert a feeding tube and initiate enteral nutrition. Enteral nutrition can usually be started during the emergent phase at low rates and increased over 24 to 48 hours to the goal rate. During the emergent phase, the patient will be unable to eat enough calories to meet nutritional needs and may have a paralytic ileus that prevents adequate nutrient absorption. Vitamins and minerals may be given during the emergent phase, but these will not assist in meeting the patient's caloric needs. Parenteral nutrition increases the infection risk, does not help preserve gastrointestinal function, and is not routinely used in burn patients unless the gastrointestinal tract is not available for use.

The patient in the emergent phase of a burn injury is being treated for severe pain. What medication should the nurse anticipate administering to the patient? a. Subcutaneous (SQ) tetanus toxoid b. Intravenous (IV) morphine sulfate c. Intramuscular (IM) hydromorphone d. Oral oxycodone and acetaminophen

b. Intravenous (IV) morphine sulfate IV medications are used for burn injuries in the emergent phase to rapidly deliver relief and prevent unpredictable absorption that would occur with the IM route. The PO route is not used because GI function is slowed or impaired because of shock or paralytic ileus, although oxycodone and acetaminophen may be used later in the patient's recovery. Tetanus toxoid may be administered but not for pain.

The nurse is planning care for the patient in the acute phase of a burn injury. What nursing action is important for the nurse to perform after the progression from the emergent to the acute phase? a. Begin IV fluid replacement. b. Monitor for signs of complications. c. Assess and manage pain and anxiety. d. Discuss possible reconstructive surgery.

b. Monitor for signs of complications. Monitoring for complications (e.g., wound infection, pneumonia, contractures) is needed in the acute phase. Fluid replacement occurs in the emergent phase. Assessing and managing pain and anxiety occurs in the emergent and the acute phases. Discussing possible reconstructive surgeries is done in the rehabilitation phase.

A patient is admitted to the burn unit with burns to the head, face, and hands. Initially, wheezes are heard, but an hour later, the lung sounds are decreased, and no wheezes are audible. What action should the nurse take? a. Encourage the patient to cough and auscultate the lungs again. b. Notify the health care provider and prepare for endotracheal intubation. c. Document the results and continue to monitor the patient's respiratory rate. d. Reposition the patient in high-Fowler's position and reassess breath sounds.

b. Notify the health care provider and prepare for endotracheal intubation. The patient's history and clinical manifestations suggest airway edema, and the health care provider should be notified at once so that intubation can be done rapidly. Placing the patient in a more upright position or having the patient cough will not address the problem of airway edema. Continuing to monitor is inappropriate because immediate action should occur.

A patient admitted with acute respiratory failure has ineffective airway clearance from thick secretions. Which nursing intervention would specifically address this patient problem? a. Encourage use of the incentive spirometer. b. Offer the patient fluids at frequent intervals. c. Teach the patient the importance of ambulation. d. Titrate oxygen level to keep O2 saturation above 93%.

b. Offer the patient fluids at frequent intervals. Because the reason for the poor airway clearance is the thick secretions, the best action will be to encourage the patient to improve oral fluid intake. Patients should be instructed to use the incentive spirometer on a regular basis (e.g., every hour) to facilitate the clearance of the secretions. The other actions may be helpful in improving the patient's gas exchange, but they do not address the thick secretions that are causing the poor airway clearance.

A patient with extensive electrical burn injuries is admitted to the emergency department. Which prescribed intervention should the nurse implement first? a. Assess pain level. b. Place on heart monitor. c. Check potassium level. d. Assess oral temperature.

b. Place on heart monitor. After an electrical burn, the patient is at risk for life-threatening dysrhythmias and should be placed on a heart monitor. Assessing the oral temperature and pain is not as important as assessing for dysrhythmias. Checking the potassium level is important, but it will take time before the laboratory results are back. The first intervention is to place the patient on a heart monitor and assess for dysrhythmias so that they can be monitored and treated if necessary.

The nurse is caring for a patient with superficial partial-thickness burns of the face sustained within the last 12 hours. Upon assessment the nurse would expect to find which manifestation? a. Blisters b. Reddening of the skin c. Destruction of all skin layers d. Damage to sebaceous glands

b. Reddening of the skin The clinical appearance of superficial partial-thickness burns includes reddening of the skin, blanching with pressure, and pain and minimal swelling with no vesicles or blistering during the first 24 hours.

The nurse educator is evaluating the performance of a new registered nurse (RN) who is providing care to a patient receiving mechanical ventilation with 15 cm H2O of peak end-expiratory pressure (PEEP). Which action indicates that the new RN is safe? a. The RN plans to suction the patient every 1 to 2 hours. b. The RN uses a closed-suction technique to suction the patient. c. The RN changes the ventilator circuit tubing routinely every 48 hours. d. The RN tapes the connection between the ventilator tubing and the ET.

b. The RN uses a closed-suction technique to suction the patient. The closed-suction technique is used when patients require high levels of PEEP (>10 cm H2O) to prevent the loss of PEEP that occurs when disconnecting the patient from the ventilator. Suctioning should not be scheduled routinely, but it should be done only when patient assessment data indicate the need for suctioning. Taping connections between the ET and ventilator tubing would restrict the ability of the tubing to swivel in response to patient repositioning. Ventilator tubing changes increase the risk for ventilator-associated pneumonia and are not indicated routinely.

A nurse is caring for a patient with ARDS who is being treated with mechanical ventilation and high levels of positive end-expiratory pressure (PEEP). Which assessment finding by the nurse indicates that the PEEP may need to be reduced? a. The patient's PaO2 is 50 mm Hg and the SaO2 is 88%. b. The patient has subcutaneous emphysema on the upper thorax. c. The patient has bronchial breath sounds in both the lung fields. d. The patient has a first-degree atrioventricular heart block with a rate of 58 beats/min.

b. The patient has subcutaneous emphysema on the upper thorax. The subcutaneous emphysema indicates barotrauma caused by positive pressure ventilation and PEEP. Bradycardia, hypoxemia, and bronchial breath sounds are all concerns and will need to be addressed, but they are not specific indications that PEEP should be reduced.

Eight hours after a thermal burn covering 50% of a patient's total body surface area (TBSA), the nurse assesses the patient. The patient weighs 92 kg (202.4 lb). Which information would be a priority to communicate to the health care provider? a. Blood pressure is 95/48 per arterial line. b. Urine output of 41 mL over past 2 hours. c. Serous exudate is leaking from the burns. d. Heart monitor shows sinus tachycardia of 108.

b. Urine output of 41 mL over past 2 hours. The urine output should be at least 0.5 to 1.0 mL/kg/hr during the emergent phase, when the patient is at great risk for hypovolemic shock. The nurse should notify the health care provider because a higher IV fluid rate is needed. BP during the emergent phase should be greater than 90 mm Hg systolic and the pulse rate should be less than 120 beats/min. Serous exudate from the burns is expected during the emergent phase.

A young adult patient who is in the rehabilitation phase after having deep partial-thickness face and neck burns has been having difficulty with body image over the past several months. Which statement by the patient best indicates that the problem is resolving? a. "I'm glad the scars are only temporary." b. "I will avoid using a pillow, so my neck will be OK." c. "Do you think dark beige makeup will cover this scar?" d. "I don't think my boyfriend will want to look at me now."

c. "Do you think dark beige makeup will cover this scar?" The willingness to use strategies to enhance appearance is an indication that the disturbed body image is resolving. Expressing feelings about the scars shows a willingness to discuss appearance but not resolution of the problem. Because deep partial-thickness burns leave permanent scars, a statement that the scars are temporary shows denial rather than resolution of the problem. Avoiding using a pillow will help prevent contractures, but it does not address the problem of disturbed body image.

A patient with severe burns has crystalloid fluid replacement ordered using the Parkland formula. The initial volume of fluid to be given in the first 24 hours is 30,000 mL. The initial rate of administration is 1875 mL/hr. After the first 8 hours, what rate should the nurse infuse the IV fluids? a. 219 mL/hr b. 625 mL/hr c. 938 mL/hr d. 1875 mL/hr

c. 938 mL/hr Half of the fluid replacement using the Parkland formula is administered in the first 8 hours and the other half over the next 16 hours. In this case, the patient should receive half of the initial rate, or 938 mL/hr.

Which patient should the nurse prepare to transfer to a regional burn center? a. A 25-yr-old pregnant patient with a carboxyhemoglobin level of 1.5% b. A 39-yr-old patient with a partial-thickness burn to the right upper arm c. A 53-yr-old patient with a chemical burn to the anterior chest and neck d. A 42-yr-old patient who is scheduled for skin grafting of a burn wound

c. A 53-yr-old patient with a chemical burn to the anterior chest and neck The American Burn Association (ABA) has established referral criteria to determine which burn injuries should be treated in burn centers where specialized facilities and personnel are available to manage this type of trauma. Patients with chemical burns should be referred to a burn center. A normal serum carboxyhemoglobin level for nonsmokers is 0% to 1.5% and for smokers is 4% to 9%. Skin grafting for burn wound management is not a criterion for a referral to a burn center. Partial-thickness burns greater than 10% total body surface area (TBSA) should be referred to a burn center. A burn to the right upper arm is 4% TBSA.

Which factor indicates that tracheostomy placement would be preferable to endotracheal intubation? a. The patient is unable to clear secretions. b. The patient is at high risk for aspiration. c. A long-term airway is probably necessary. d. An upper airway obstruction is impairing the patient's ventilation.

c. A long-term airway is probably necessary. A tracheostomy is indicated when the need for an artificial airway is expected to be long term. Aspiration risk, an inability to clear secretions, and upper airway obstruction are indications for an artificial airway, but these are not specific indications for tracheostomy.

Which patient is most appropriate for the burn unit charge nurse to assign to a registered nurse (RN) who has floated from the hospital medical unit? a. A patient who has twice-daily burn debridements to partial-thickness facial burns. b. A patient who just returned from having a cultured epithelial autograft to the chest. c. A patient who has a 15% weight loss from admission and will need enteral feedings. d. A patient who has blebs under an autograft on the thigh and has an order for bleb aspiration.

c. A patient who has a 15% weight loss from admission and will need enteral feedings. An RN from a medical unit would be familiar with malnutrition and with administration and evaluation of response to enteral feedings. The other patients need burn assessment and care that is more appropriate for staff who regularly care for burned patients

The nurse notes thick, white secretions in the endotracheal tube (ET) of a patient who is receiving mechanical ventilation. Which intervention will most directly treat this finding? a. Reposition the patient every 1 to 2 hours. b. Increase suctioning frequency to every hour. c. Add additional water to the patient's enteral feedings. d. Instill 5 mL of sterile saline into the ET before suctioning.

c. Add additional water to the patient's enteral feedings. Because the patient's secretions are thick, better hydration is indicated. Suctioning every hour without any specific evidence for the need will increase the incidence of mucosal trauma and would not address the etiology of the ineffective airway clearance. Instillation of saline does not liquefy secretions and may decrease the SpO2. Repositioning the patient is appropriate but will not decrease the thickness of secretions.

The nurse is admitting a patient with asthma in acute respiratory distress. The nurse auscultates the patient's lungs and notes cessation of the inspiratory wheezing. The patient has not yet received any medication. What should this finding suggest to the nurse? a. Spontaneous resolution of the acute asthma attack b. An acute development of bilateral pleural effusions c. Airway constriction requiring immediate interventions d. Overworked intercostal muscles resulting in poor air exchange

c. Airway constriction requiring immediate interventions When a patient in respiratory distress has inspiratory wheezing and then it ceases, it is an indication of airway obstruction. This finding requires emergency action to restore airway patency. Cessation of inspiratory wheezing does not indicate spontaneous resolution of the acute asthma attack, bilateral pleural effusion development, or overworked intercostal muscles in this asthmatic patient that is in acute respiratory distress.

Which diagnostic test will provide the nurse with the most specific information to evaluate the effectiveness of interventions for a patient with ventilatory failure? a. Chest x-ray b. O2 saturation c. Arterial blood gas analysis d. Central venous pressure monitoring

c. Arterial blood gas analysis Arterial blood gas (ABG) analysis is most useful in this setting because ventilatory failure causes problems with CO2 retention, and ABGs give information about the PaCO2 and pH. The other tests may also be done to help in assessing oxygenation or determining the cause of the patient's ventilatory failure.

The nurse is caring for a patient with multiple musculoskeletal injuries who has developed acute respiratory distress syndrome (ARDS). Which intervention should the nurse initiate to prevent stress ulcers? a. Observe stools for frank bleeding and occult blood. b. Maintain head of the bed elevation at 30 to 45 degrees. c. Begin enteral feedings as soon as bowel sounds are present. d. Administer prescribed lorazepam (Ativan) to reduce anxiety.

c. Begin enteral feedings as soon as bowel sounds are present. Stress ulcers prevention includes early initiation of enteral nutrition to protect the gastrointestinal (GI) tract from mucosal damage. Monitoring for GI bleeding does not prevent stress ulcers. Ventilator-associated pneumonia related to aspiration is prevented by elevation of the head of bed to 30 to 45 degrees Stress ulcers are not caused by anxiety. Stress ulcers are related to GI ischemia from hypotension, shock, and acidosis.

To evaluate the effectiveness of the pantoprazole (Protonix) given to a patient with systemic inflammatory response syndrome (SIRS), which assessment will the nurse perform? a. Auscultate bowel sounds. b. Ask the patient about nausea. c. Check stools for occult blood. d. Palpate for abdominal tenderness.

c. Check stools for occult blood. Proton pump inhibitors are given to decrease the risk for stress ulcers in critically ill patients. The other assessments will also be done, but these will not help in determining the effectiveness of the pantoprazole administration.

A patient has just arrived in the emergency department after an electrical burn from exposure to a high-voltage current. What is the priority nursing assessment? a. Oral temperature b. Peripheral pulses c. Extremity movement d. Pupil reaction to light

c. Extremity movement All patients with electrical burns should be considered at risk for cervical spine injury, and assessment of extremity movement will provide baseline data. The other assessment data are necessary but not as essential as determining the cervical spine status.

The nurse is caring for a patient with multiple fractured ribs from a motor vehicle crash. Which assessment findings would be early indications that the patient is developing respiratory failure? a. Tachycardia and pursed lip breathing b. Kussmaul respirations and hypotension c. Frequent position changes and agitation d. Cyanosis and increased capillary refill time

c. Frequent position changes and agitation A change in mental status is an early indication of respiratory failure. The brain is sensitive to variations in oxygenation, arterial carbon dioxide levels, and acid-base balance. Restlessness, confusion, agitation, and combative behavior suggest inadequate oxygen delivery to the brain.

On admission to the burn unit, a patient with an approximate 25% total body surface area (TBSA) burn has the following initial laboratory results: Hct 58%, Hgb 18.2 mg/dL (172 g/L), serum K+ 4.9 mEq/L (4.8 mmol/L), and serum Na+ 135 mEq/L (135 mmol/L). Which of the following prescribed actions should be the nurse's priority? a. Monitoring urine output every 4 hours b. Continuing to monitor the laboratory results c. Increasing the rate of the ordered IV solution d. Typing and crossmatching for a blood transfusion

c. Increasing the rate of the ordered IV solution The patient's laboratory results show hemoconcentration, which may lead to a decrease in blood flow to the microcirculation unless fluid intake is increased. Because the hematocrit and hemoglobin are elevated, a transfusion is inappropriate, although transfusions may be needed after the emergent phase once the patient's fluid balance has been restored. On admission to a burn unit, the urine output would be monitored more often than every 4 hours (likely every hour).

When caring for a patient with an electrical burn injury, which order from the health care provider should the nurse question? a. Mannitol 75 gram IV b. Urine for myoglobulin c. Lactated Ringer's solution at 25 mL/hr d. Sodium bicarbonate 24 mEq every 4 hours

c. Lactated Ringer's solution at 25 mL/hr Electrical injury puts the patient at risk for myoglobinuria, which can lead to acute renal tubular necrosis (ATN). Treatment consists of infusing lactated Ringer's solution at 2 to 4 mL/kg/%TBSA, a rate sufficient to maintain urinary output at 75 to 100 mL/hr. Mannitol can also be used to maintain urine output. Sodium bicarbonate may be given to alkalinize the urine. The urine would also be monitored for the presence of myoglobin. An infusion rate of 25 mL/hr is not sufficient to maintain adequate urine output in prevention and treatment of ATN.

A nurse is caring for a patient with second- and third-degree burns to 50% of the body. The nurse prepares fluid resuscitation based on knowledge of the Parkland (Baxter) formula that includes which recommendation? a. The total 24-hour fluid requirement should be administered in the first 8 hours. b. One half of the total 24-hour fluid requirement should be administered in the first 4 hours. c. One half of the total 24-hour fluid requirement should be administered in the first 8 hours. d. One third of the total 24-hour fluid requirement should be administered in the first 4 hours.

c. One half of the total 24-hour fluid requirement should be administered in the first 8 hours. Fluid resuscitation with the Parkland (Baxter) formula recommends that one half of the total fluid requirement should be administered in the first 8 hours, one quarter of total fluid requirement should be administered in the second 8 hours, and one quarter of total fluid requirement should be administered in the third 8 hours.

The nurse is caring for a patient who was admitted 1 week ago with multiple rib fractures, pulmonary contusions, and a left femur fracture from a motor vehicle crash. The provider states the patient has developed sepsis, and the family members have many questions. Which information should the nurse include when explaining the early stage of sepsis? a. Weaning the patient from the ventilator is the top priority in sepsis. b. Antibiotics are not useful when an infection has progressed to sepsis. c. Large amounts of IV fluid are required in sepsis to fill dilated blood vessels. d. The patient has recovered from sepsis if he has warm skin and ruddy cheeks.

c. Large amounts of IV fluid are required in sepsis to fill dilated blood vessels. Patients with sepsis may be normovolemic, but because of acute vasodilation, relative hypovolemia and hypotension occur. Patients in septic shock require large amounts of fluid replacement and may require frequent fluid boluses to maintain circulation. Antibiotics are an important component of therapy for patients with septic shock. They should be started after cultures (e.g., blood, urine) are obtained and within the first hour of septic shock. Oxygenating the tissues is the top priority in sepsis, so efforts to wean septic patients from mechanical ventilation halt until sepsis is resolving. Additional respiratory support may be needed during sepsis. Although cool and clammy skin is present in other early shock states, the patient in early septic shock may feel warm and flushed because of a hyperdynamic state.

The nurse notes that a patient's endotracheal tube (ET), which was at the 22-cm mark, is now at the 25-cm mark, and the patient is anxious and restless. Which action should the nurse take next? a. Check the O2 saturation. b. Offer reassurance to the patient. c. Listen to the patient's breath sounds. d. Notify the patient's health care provider.

c. Listen to the patient's breath sounds. The nurse should first determine whether the ET tube has been displaced into the right mainstem bronchus by listening for unilateral breath sounds. If so, assistance will be needed to reposition the tube immediately. The other actions are also appropriate, but detection and correction of tube malposition are the most critical actions.

A patient with type 2 diabetes is in the acute phase of burn care with electrical burns on the left side of the body and a serum glucose level of 485 mg/dL. What is the nurse's priority intervention for this patient? a. Replace the blood lost. b. Maintain a neutral pH. c. Maintain fluid balance. d. Replace serum potassium.

c. Maintain fluid balance. This patient most likely has hyperosmolar hyperglycemic syndrome (HHS). HHS dehydrates a patient rapidly. HHS combined with the massive fluid losses of a burn tremendously increase this patient's risk for hypovolemic shock and serious hypotension. This is clearly the nurse's priority because the nurse must keep up with the patient's fluid requirements to prevent circulatory collapse caused by low intravascular volume. There is no mention of blood loss. Fluid resuscitation will help to correct the pH and serum potassium abnormalities.

After a patient who has septic shock receives 2 L of normal saline intravenously, the central venous pressure is 10 mm Hg and the blood pressure is 82/40 mm Hg. What medication should the nurse anticipate? a. Furosemide b. Nitroglycerin c. Norepinephrine d. Sodium nitroprusside

c. Norepinephrine When fluid resuscitation is unsuccessful, vasopressor drugs are given to increase the systemic vascular resistance (SVR) and blood pressure and improve tissue perfusion. Furosemide would cause diuresis and further decrease the BP. Nitroglycerin would decrease the preload and further drop cardiac output and BP. Nitroprusside is an arterial vasodilator and would further decrease SVR.

A patient's localized infection has become systemic and septic shock is suspected. What medication would be given to treat septic shock refractory to fluids? a. Insulin infusion b. Furosemide IV push c. Norepinephrine administered by titration d. Administration of nitrates and β-adrenergic blockers

c. Norepinephrine administered by titration If fluid resuscitation using crystalloids is not effective, vasopressor medications, such as norepinephrine (Levophed) and dopamine, are indicated to restore mean arterial pressure (MAP). Nitrates and β-adrenergic blockers are most often used in the treatment of patients in cardiogenic shock. Furosemide (Lasix) is indicated for patients with fluid volume overload. Insulin infusion may be given to normalize blood sugar and improve overall outcomes, but it is not considered a medication used to treat shock.

The nurse assesses vital signs for a patient admitted 2 days ago with gram-negative sepsis: temperature of 101.2° F, blood pressure of 90/56 mm Hg, pulse of 92 beats/min, and respirations of 34 breaths/min. Which action should the nurse take next? a. Give the scheduled IV antibiotic. b. Give the PRN acetaminophen (Tylenol). c. Obtain oxygen saturation using pulse oximetry. d. Notify the health care provider of the patient's vital signs.

c. Obtain oxygen saturation using pulse oximetry. The patient's increased respiratory rate in combination with the admission diagnosis of gram-negative sepsis indicates that acute respiratory distress syndrome (ARDS) may be developing. The nurse should check for hypoxemia, a hallmark of ARDS. The health care provider should be notified after further assessment of the patient. Giving the scheduled antibiotic and the PRN acetaminophen will also be done, but they are not the highest priority for a patient who may be developing ARDS.

The nurse is caring for an older patient who was hospitalized 2 days earlier with community-acquired pneumonia. Which assessment information is most important to communicate to the health care provider? a. Persistent cough of blood-tinged sputum. b. Scattered crackles in the posterior lung bases. c. Oxygen saturation 90% on 100% O2 by non-rebreather mask. d. Temperature 101.5° F (38.6° C) after 2 days of IV antibiotics.

c. Oxygen saturation 90% on 100% O2 by non-rebreather mask. The patient's low SpO2 despite receiving a high fraction of inspired oxygen (FIO2) indicates the possibility of acute respiratory distress syndrome (ARDS). The patient's blood-tinged sputum and scattered crackles are not unusual in a patient with pneumonia, although they do need continued monitoring. The continued temperature elevation indicates a possible need to change antibiotics, but this is not as urgent a concern as the progression toward hypoxemia despite an increase in O2 flowrate.

After change-of-shift report on a ventilator weaning unit, which patient should the nurse assess first? a. Patient who failed a spontaneous breathing trial and has been placed in a rest mode on the ventilator b. Patient who is intubated and has continuous partial pressure end-tidal CO2 (PETCO2) monitoring c. Patient who was successfully weaned and extubated 4 hours ago and has no urine output for the last 6 hours d. Patient with a central venous O2 saturation (ScvO2) of 69% while on bilevel positive airway pressure (BiPAP)

c. Patient who was successfully weaned and extubated 4 hours ago and has no urine output for the last 6 hours The decreased urine output may indicate acute kidney injury or that the patient's cardiac output and perfusion of vital organs have decreased. Any of these causes would require rapid action. The data about the other patients indicate that their conditions are stable and do not require immediate assessment or changes in their care. Continuous PETCO2 monitoring is frequently used when patients are intubated. The rest mode should be used to allow patient recovery after a failed SBT. A ScvO2 of 69% is within normal limits.

After change-of-shift report in the progressive care unit, who should the nurse care for first? a. Patient who had an inferior myocardial infarction 2 days ago and has crackles in the lung bases. b. Patient who had a T5 spinal cord injury 1 week ago and currently has a heart rate of 54 beats/min. c. Patient with suspected urosepsis who has new prescriptions for urine and blood cultures and antibiotics. d. Patient admitted with anaphylaxis 3 hours ago who has clear lung sounds and a blood pressure of 108/58 mm Hg.

c. Patient with suspected urosepsis who has new prescriptions for urine and blood cultures and antibiotics. Antibiotics should be given within the first hour for patients who have sepsis or suspected sepsis to prevent progression to systemic inflammatory response syndrome and septic shock. The data on the other patients indicate that they are more stable. Crackles heard only at the lung bases do not need immediate intervention in a patient who has had a myocardial infarction. Mild bradycardia does not usually need treatment in patients with a spinal cord injury. The findings for the patient admitted with anaphylaxis show resolution of bronchospasm and hypotension.

When planning care for a patient on a mechanical ventilator, the application of positive end-expiratory pressure (PEEP) to the ventilator settings has which therapeutic effect? a. Increased inflation of the lungs b. Prevention of barotrauma to the lung tissue c. Prevention of alveolar collapse during expiration d. Increased fraction of inspired oxygen concentration (FIO2) administration

c. Prevention of alveolar collapse during expiration PEEP is positive pressure that is applied to the airway during exhalation. This positive pressure prevents the alveoli from collapsing, improving oxygenation and enabling a reduced FIO2 requirement. PEEP does not cause increased inflation of the lungs or prevent barotrauma. Auto-PEEP resulting from inadequate exhalation time may contribute to barotrauma.

The nurse is caring for a patient with acute decompensated heart failure who has a pulmonary artery catheter. Which assessment indicates the patient's condition is improving? a. Cardiac output (CO) is 3.5 L/min. b. Central venous pressure (CVP) is 10 mm Hg. c. Pulmonary artery wedge pressure (PAWP) is 10 mm Hg. d. Systemic vascular resistance (SVR) is 1500 dynes/sec/cm−5.

c. Pulmonary artery wedge pressure (PAWP) is 10 mm Hg. PAWP is the most sensitive indicator of cardiac function and fluid volume status. Normal range for PAWP is 6 to 12 mm Hg. PAWP is increased in heart failure. Normal range for CVP is 2 to 8 mm Hg. An elevated CVP indicates right-sided heart failure or volume overload. CO is decreased in heart failure. Normal cardiac output is 4 to 8 L/minute. SVR is increased in left-sided heart failure. Normal SVR is 800 to 1200 dynes/sec/cm−5.

The patient received a cultured epithelial autograft (CEA) to the entire left leg. What should the nurse include in the discharge teaching for this patient? a. Sit or lie in the position of comfort. b. Wear a pressure garment for 8 hours each day. c. Refer the patient to a counselor for psychosocial support. d. Use the sun to increase the skin color on the healed areas.

c. Refer the patient to a counselor for psychosocial support. In the rehabilitation phase, the patient will work toward resuming a functional role in society, but frequently there are body image concerns and grieving for the loss of the way the patient looked and functioned before the burn, so continued counseling helps the patient in this phase as well. Putting the leg in the position of comfort is more likely to lead to contractures than to help the patient. If a pressure garment is prescribed, it is used for 24 hours/day for as long as 12 to 18 months. Sunlight should be avoided to prevent injury, and sunscreen should always be worn when the patient is outside.

The nurse is weaning a 68-kg patient who has chronic obstructive pulmonary disease (COPD) from mechanical ventilation. Which finding indicates that the weaning protocol should be stopped? a. The patient's heart rate is 97 beats/min. b. The patient's oxygen saturation is 93%. c. The patient respiratory rate is 32 breaths/min. d. The patient's spontaneous tidal volume is 450 mL.

c. The patient respiratory rate is 32 breaths/min. Tachypnea is a sign that the patient's work of breathing is too high to allow weaning to proceed. The patient's heart rate is within normal limits, but the nurse should continue to monitor it. An O2 saturation of 93% is acceptable for a patient with COPD. A spontaneous tidal volume of 450 mL is within the acceptable range.

A patient with chronic obstructive pulmonary disease (COPD) arrives in the emergency department reporting shortness of breath on minimal exertion. Which assessment finding by the nurse is most important to report to the health care provider? a. The patient has bibasilar lung crackles. b. The patient is sitting in the tripod position. c. The patient's respiratory rate is 10 breaths/min. d. The patient's pulse oximetry shows a 91% O2 saturation.

c. The patient's respiratory rate is 10 breaths/min. A drop in respiratory rate in a patient with respiratory distress suggests the onset of fatigue and a high risk for respiratory arrest. Therefore, immediate action such as positive-pressure ventilation is needed. Patients who are experiencing respiratory distress frequently sit in the tripod position because it decreases the work of breathing. Crackles in the lung bases may be the baseline for a patient with COPD. An O2 saturation of 91% is common in patients with COPD and will provide adequate gas exchange and tissue oxygenation.

Which assessment finding by the nurse caring for a patient receiving mechanical ventilation indicates the need for suctioning? a. The patient was last suctioned 6 hours ago. b. The patient's oxygen saturation drops to 93%. c. The patient's respiratory rate is 32 breaths/min. d. The patient has occasional audible expiratory wheezes.

c. The patient's respiratory rate is 32 breaths/min. The increase in respiratory rate indicates that the patient may have decreased airway clearance and requires suctioning. Suctioning is done when patient assessment data indicate that it is needed and not on a scheduled basis. Occasional expiratory wheezes do not indicate poor airway clearance. Suctioning the patient may induce bronchospasm and increase wheezing. An O2 saturation of 93% is acceptable and does not suggest that immediate suctioning is needed.

A patient with burns covering 40% total body surface area (TBSA) is in the acute phase of burn treatment. Which snack would be best for the nurse to offer to this patient? a. Bananas b. Orange gelatin c. Vanilla milkshake d. Whole grain bagel

c. Vanilla milkshake A patient with a burn injury needs high-protein and high-calorie food intake, and the milkshake is the highest in these nutrients. The other choices are not as nutrient dense as the milkshake. Gelatin is likely high in sugar. The bagel is a good carbohydrate choice but low in protein. Bananas are a good source of potassium but are not high in protein and calories.

The nurse is assisting in the care of several patients in the critical care unit. Which patient is most at risk for developing multiple organ dysfunction syndrome (MODS)? a. A 22-yr-old patient with systemic lupus erythematosus admitted with a pelvic fracture b. A 48-yr-old patient with lung cancer admitted for syndrome of inappropriate antidiuretic hormone and hyponatremia c. A 65-yr-old patient with coronary artery disease, dyslipidemia, and primary hypertension admitted for unstable angina d. A 82-yr-old patient with type 2 diabetes and chronic kidney disease admitted for peritonitis from a peritoneal dialysis catheter infection

d. A 82-yr-old patient with type 2 diabetes and chronic kidney disease admitted for peritonitis from a peritoneal dialysis catheter infection A patient with peritonitis is at high risk for developing sepsis. In addition, a patient with diabetes is at high risk for infections and impaired healing. Sepsis and septic shock are the most common causes of MODS. Those at greatest risk for developing MODS are older adults and persons with significant tissue injury or preexisting disease. MODS can be initiated by any severe injury or disease process that activates a massive systemic inflammatory response.

Which patient would most benefit from noninvasive positive pressure ventilation (NIPPV) to promote oxygenation? a. A patient whose cardiac output and blood pressure are unstable. b. A patient with cystic fibrosis who is currently producing copious secretions. c. A patient with respiratory failure due to a head injury with loss of consciousness. d. A patient who has respiratory failure because of the progression of myasthenia gravis.

d. A patient who has respiratory failure because of the progression of myasthenia gravis. NIPPV such as continuous positive airway pressure (CPAP) is most effective in treating patients with respiratory failure resulting from chest wall and neuromuscular disease. It is not recommended in patients who are experiencing hemodynamic instability, decreased level of consciousness, or excessive secretions.

After receiving change-of-shift report on a medical unit, which patient should the nurse assess first? a. A patient with cystic fibrosis who has thick, green-colored sputum. b. A patient with pneumonia who has crackles bilaterally in the lung bases. c. A patient with emphysema who has an oxygen saturation of 90% to 92%. d. A patient with septicemia who has intercostal and suprasternal retractions.

d. A patient with septicemia who has intercostal and suprasternal retractions. This patient's history of septicemia and labored breathing suggest the onset of ARDS, which will require rapid interventions such as administration of O2 and use of positive-pressure ventilation. The other patients should also be assessed, but their assessment data are typical of their disease processes and do not suggest deterioration in their status.

Which action will the nurse include in the plan of care for a patient in the rehabilitation phase after a burn injury to the right arm and chest? a. Keep the right arm in a position of comfort. b. Avoid the use of sustained-release narcotics. c. Teach about the purpose of tetanus immunization. d. Apply water-based cream to burned areas frequently.

d. Apply water-based cream to burned areas frequently. Application of water-based emollients will moisturize new skin and decrease flakiness and itching. To avoid contractures, the joints of the right arm should be positioned in an extended position, which is not the position of comfort. Patients may need to continue the use of opioids during rehabilitation. Tetanus immunization would have been given during the emergent phase of the burn injury.

The nurse observes a new onset of agitation and confusion in a patient with chronic obstructive pulmonary disease (COPD). Which action should the nurse take first? a. Observe for facial symmetry. b. Notify the health care provider. c. Attempt to calm and reorient the patient. d. Assess oxygenation using pulse oximetry.

d. Assess oxygenation using pulse oximetry. Because agitation and confusion are often the initial indicators of hypoxemia, the nurse's initial action should be to assess O2 saturation. The other actions are appropriate, but assessment of oxygenation takes priority over other assessments and notification of the health care provider.

Four hours after mechanical ventilation is initiated, a patient's arterial blood gas (ABG) results include a pH of 7.51, PaO2 of 82 mm Hg, PaCO2 of 26 mm Hg, and HCO3 of 23 mEq/L (23 mmol/L). What change should the nurse anticipate to the ventilator settings? a. Increase the FIO2. b. Increase the tidal volume. c. Increase the respiratory rate. d. Decrease the respiratory rate.

d. Decrease the respiratory rate. The patient's PaCO2 and pH indicate respiratory alkalosis caused by too high a respiratory rate. The PaO2 is appropriate for a patient with COPD and increasing the respiratory rate and tidal volume would further lower the PaCO2.

In caring for a patient with burns to the back, the nurse knows that the patient is moving out of the emergent phase of burn injury when what is observed? a. Serum sodium and potassium increase. b. Serum sodium and potassium decrease. c. Edema and arterial blood gases improve. d. Diuresis occurs and hematocrit decreases.

d. Diuresis occurs and hematocrit decreases. Toward the end of the emergent phase, fluid loss and edema formation end. Interstitial fluid returns to the vascular space and diuresis occurs. Urinary output is the most commonly used parameter to assess the adequacy of fluid resuscitation. The hemolysis of red blood cells (RBCs) and thrombosis of burned capillaries also decreases circulating RBCs. When the fluid balance has been restored, dilution causes the hematocrit levels to drop. Initially sodium moves to the interstitial spaces and remains there until edema formation ceases, so sodium levels increase at the end of the emergent phase as the sodium moves back to the vasculature. Initially potassium level increases as it is released from injured cells and hemolyzed RBCs, so potassium levels decrease at the end of the emergent phase when fluid levels normalize.

A patient with sepsis is orally intubated on mechanical ventilation. Which nursing action is most important? a. Giving morphine for discomfort b. Clustering nursing care activities c. Using an open-suctioning technique d. Elevating the head of the bed 30 degrees

d. Elevating the head of the bed 30 degrees The major complications of endotracheal intubation are unplanned extubation and aspiration. To prevent aspiration, all intubated patients and patients receiving enteral feedings must have the head of the bed elevated a minimum of 30 to 45 degrees unless medically contraindicated. Closed-suction technique is preferred because oxygenation and ventilation are maintained during suctioning, and exposure to secretions is reduced. The nurse should provide comfort measures, such as morphine, to relieve anxiety and pain associated with intubation. To promote rest and sleep, the nurse should limit noise and cluster activities.

The nurse is caring for a patient with partial- and full-thickness burns to 65% of the body. When planning nutritional interventions for this patient, what dietary choices should the nurse implement? a. Full liquids only b. Whatever the patient requests c. High-protein and low-sodium foods d. High-calorie and high-protein foods

d. High-calorie and high-protein foods A hypermetabolic state occurs proportional to the size of the burn area. Massive catabolism can occur and is characterized by protein breakdown and increased gluconeogenesis. Caloric needs are often in the 5000-kcal range. Failure to supply adequate calories and protein leads to malnutrition and delayed healing.

The nurse is caring for a patient with a subarachnoid hemorrhage who is intubated and placed on a mechanical ventilator with 10 cm H2O of peak end-expiratory pressure (PEEP). What new finding indicates that the nurse needs to notify the health care provider immediately? a. O2 saturation of 93% b. Respirations of 20 breaths/min c. Green nasogastric tube drainage d. Increased jugular venous distention

d. Increased jugular venous distention Increases in jugular venous distention in a patient with a subarachnoid hemorrhage may indicate an increase in intracranial pressure (ICP) and that the PEEP setting is too high for this patient. A respiratory rate of 20, O2 saturation of 93%, and green nasogastric tube drainage are within normal limits.

How should the nurse maintain proper cuff pressure of an endotracheal tube (ET) when the patient is on mechanical ventilation? a. Inflate the cuff with a minimum of 10 mL of air. b. Inflate the cuff until the pilot balloon is firm on palpation. c. Inject air into the cuff until a manometer shows 15 mm Hg pressure. d. Inject air into the cuff until a slight leak is heard only at peak inflation.

d. Inject air into the cuff until a slight leak is heard only at peak inflation. The minimal occluding volume technique involves injecting air into the cuff until an air leak is present only at peak inflation. The volume to inflate the cuff varies with the ET and the patient's size. Cuff pressure should be maintained at 20 to 25 mm Hg. An accurate assessment of cuff pressure cannot be obtained by palpating the pilot balloon.

Which nursing interventions included in the care of a mechanically ventilated patient with acute respiratory failure can the registered nurse (RN) delegate to an experienced licensed practical/vocational nurse (LPN/VN) working in the intensive care unit? a. Assess breath sounds every hour. b. Monitor central venous pressures. c. Place patient in the prone position. d. Insert an indwelling urinary catheter.

d. Insert an indwelling urinary catheter. Insertion of indwelling urinary catheters is included in LPN/VN education and scope of practice and can be safely delegated to an LPN/VN who is experienced in caring for critically ill patients. Placing a patient who is on a ventilator in the prone position requires multiple staff and should be supervised by an RN. Assessment of breath sounds and obtaining central venous pressures require advanced assessment skills and should be done by the RN caring for a critically ill patient.

A patient develops increasing dyspnea and hypoxemia 2 days after heart surgery. What procedure should the nurse anticipate assisting with to determine whether the patient has acute respiratory distress syndrome (ARDS) or pulmonary edema caused by heart failure? a. Obtaining a ventilation-perfusion scan b. Drawing blood for arterial blood gases c. Positioning the patient for a chest x-ray d. Insertion of a pulmonary artery catheter

d. Insertion of a pulmonary artery catheter Pulmonary artery wedge pressures are normal in the patient with ARDS because the fluid in the alveoli is caused by increased permeability of the alveolar-capillary membrane rather than by the backup of fluid from the lungs (as occurs in cardiogenic pulmonary edema). The other tests will not help in differentiating cardiogenic from noncardiogenic pulmonary edema.

A patient with acute respiratory distress syndrome (ARDS) who is intubated and receiving mechanical ventilation develops a right pneumothorax. Which collaborative action will the nurse anticipate next? a. Increase the tidal volume and respiratory rate. b. Decrease the fraction of inspired oxygen (FIO2). c. Perform endotracheal suctioning more frequently. d. Lower the positive end-expiratory pressure (PEEP).

d. Lower the positive end-expiratory pressure (PEEP). Because barotrauma is associated with high airway pressures, the level of PEEP should be decreased. The other actions will not decrease the risk for another pneumothorax.

During the emergent phase of burn care, which assessment is most useful in determining whether the patient is receiving adequate fluid infusion? a. Check skin turgor. b. Monitor daily weight. c. Assess mucous membranes. d. Measure hourly urine output.

d. Measure hourly urine output. When fluid intake is adequate, the urine output will be at least 0.5 to 1 mL/kg/hr. The patient's weight is not useful in this situation because of the effects of third spacing and evaporative fluid loss. Mucous membrane assessment and skin turgor also may be used, but they are not as adequate in determining that fluid infusions are maintaining adequate perfusion.

The nurse is caring for a patient intubated and on a mechanical ventilator for several days. Which weaning parameter would tell the nurse if the patient has enough muscle strength to breathe without assistance? a. Tidal volume b. Minute ventilation c. Forced vital capacity d. Negative inspiratory force

d. Negative inspiratory force The negative inspiratory force measures inspiratory muscle strength. Tidal volume and minute ventilation assess the patient's respiratory endurance. Forced vital capacity is not used as a measure to determine weaning from a ventilator.

A patient arrives in the emergency department after sustaining a full-thickness thermal burn to both arms while putting lighter fluid on a grill. What manifestations should the nurse expect? a. Severe pain, blisters, and blanching with pressure b. Pain, minimal edema, and blanching with pressure c. Redness, evidence of inhalation injury, and charred skin d. No pain, waxy white skin, and no blanching with pressure

d. No pain, waxy white skin, and no blanching with pressure With full-thickness burns, the nerves and vasculature in the dermis are destroyed so there is no pain, the tissue is dry and waxy-looking or may be charred, and there is no blanching with pressure. Severe pain, blisters, and blanching occur with partial-thickness (deep, second-degree) burns. Pain, minimal edema, blanching, and redness occur with partial-thickness (superficial, first-degree) burns.

The nurse notes premature ventricular contractions (PVCs) while suctioning a patient's endotracheal tube. Which next action by the nurse is indicated? a. Plan to suction the patient more frequently. b. Decrease the suction pressure to 80 mm Hg. c. Give antidysrhythmic medications per protocol. d. Stop and ventilate the patient with 100% oxygen.

d. Stop and ventilate the patient with 100% oxygen. Dysrhythmias during suctioning may indicate hypoxemia or sympathetic nervous system stimulation. The nurse should stop suctioning and ventilate the patient with 100% O2. There is no indication that more frequent suctioning is needed. Lowering the suction pressure will decrease the effectiveness of suctioning without improving the hypoxemia. Because the PVCs occurred during suctioning, there is no need for antidysrhythmic medications (which may have adverse effects) unless they recur when the suctioning is stopped, and patient is well oxygenated.

The charge nurse is evaluating the care that a new registered nurse (RN) provides to a patient receiving mechanical ventilation. Which action by the new RN indicates the need for more education? a. The RN increases the FIO2 to 100% before suctioning. b. The RN secures a bite block in place using adhesive tape. c. The RN asks for assistance to resecure the endotracheal tube. d. The RN positions the patient with the head of bed at 10 degrees.

d. The RN positions the patient with the head of bed at 10 degrees. The head of the patient's bed should be positioned at 30 to 45 degrees to prevent ventilator-associated pneumonia. The other actions by the new RN are appropriate.

Which finding about a patient who is receiving vasopressin to treat septic shock indicates an immediate need for the nurse to contact the health care provider? a. The patient's urine output is 18 mL/hr. b. The patient's heart rate is 110 beats/min. c. The patient's peripheral pulses are weak. d. The patient reports diffuse chest pressure.

d. The patient reports diffuse chest pressure. Because vasopressin is a potent vasoconstrictor, it may decrease coronary artery perfusion and cause chest pain or pressure. Low urine output, weal pulses, and tachycardia are consistent with the patient's diagnosis. They and should be reported to the health care provider but do not require an immediate need for a change in therapy.

Which hematologic problem significantly increases the risks associated with pulmonary artery (PA) catheter insertion? a. Leukocytosis b. Hypovolemia c. Hemolytic anemia d. Thrombocytopenia

d. Thrombocytopenia PA catheter insertion carries a significant risk of bleeding, which is increased when the patient has low levels of platelets. Leukocytosis, hypovolemia, and anemia are less likely to directly increase the risks associated with PA insertion.

A patient who has burns on the arms, legs, and chest from a house fire has become agitated and restless 8 hours after being admitted to the hospital. Which action should the nurse take first? a. Stay at the bedside and reassure the patient. b. Administer the ordered morphine sulfate IV. c. Assess orientation and level of consciousness. d. Use pulse oximetry to check oxygen saturation.

d. Use pulse oximetry to check oxygen saturation. Agitation in a patient who may have suffered inhalation injury might indicate hypoxia, and this should be assessed by the nurse first. Administration of morphine may be indicated if the nurse determines that the agitation is caused by pain. Assessing level of consciousness and orientation is appropriate but not as essential as determining whether the patient is hypoxemic. Reassurance is not helpful to reduce agitation in a hypoxemic patient.


Kaugnay na mga set ng pag-aaral

Circumference and Arc Length Assignment 100%

View Set

Econ 201M - Quiz 7: Chapter 24 Long Run Economic Growth

View Set

Milady Ch. 8 Skin Disorders and Diseases

View Set

APUSH First Semester Study Guide

View Set

Chapter 3: Ethics and Privacy Issues

View Set

Texas Promulgated, Law of Agency, National Real Estate Exam, TX: Real Estate Test Practice - NON MC

View Set